Martin Koci, Jason Alvis (Eds.): Transforming the Theological Turn: Phenomenology with Emmanuel Falque, Rowman & Littlefield, 2020

Transforming the Theological Turn: Phenomenology with Emmanuel Falque Book Cover Transforming the Theological Turn: Phenomenology with Emmanuel Falque
Martin Koci, Jason Alvis (Eds.)
Rowman & Littlefield
2020
Hardback £85.00
214

Gregory P. Floyd, Stephanie Rumpza (Eds.): The Catholic Reception of Continental Philosophy in North America, University of Toronto Press, 2020

The Catholic Reception of Continental Philosophy in North America Book Cover The Catholic Reception of Continental Philosophy in North America
Gregory P. Floyd, Stephanie Rumpza (Eds.):
University of Toronto Press
2020
Cloth $60.00
346

G. Anthony Bruno (Ed.): Schelling’s Philosophy: Freedom, Nature, and Systematicity, Oxford University Press, 2020

Schelling's Philosophy: Freedom, Nature, and Systematicity Book Cover Schelling's Philosophy: Freedom, Nature, and Systematicity
G. Anthony Bruno (Ed.)
Oxford University Press
2020
Hardback £55.00
256

Martin Heidegger: Méditation, Gallimard, 2019

Méditation Book Cover Méditation
Bibliothèque de Philosophie, Série Œuvres de Martin Heidegger
Martin Heidegger. Translated by Alain Boutot
Gallimard
2019
Paperback 32,00 €
448

Matthew Burch, Jack Marsh, Irene McMullin (Eds.): Normativity, Meaning, and the Promise of Phenomenology

Normativity, Meaning, and the Promise of Phenomenology Book Cover Normativity, Meaning, and the Promise of Phenomenology
Routledge Research in Phenomenology
Matthew Burch, Jack Marsh, Irene McMullin (Eds.)
Routledge
2019
Hardback £120.00
358

Reviewed by: Veronica Cibotaru (Universite Paris-Sorbonne / Bergische Universitaet Wuppertal)

Introduction

The purpose of Normativity, Meaning, and the Promise of Phenomenology is to study the way in which phenomenology addresses the multiple connections between normativity and meaning. The content of the book is based on a fundamental presupposition, namely, that the structure of meaning is normative. This thesis is grounded on the phenomenological studies started by Husserl and in this spirit the book explores from different points of view the structure of meaning and its conditions of possibility.

Since the authors of this book attribute this thesis directly to the views of Steven Crowell, all the articles present themselves as an explicit dialogue with Crowell’s work, to wit, Husserl, Heidegger, and the Space of Meaning: Paths Toward Transcendental Phenomenology (2001), and Normativity and Phenomenology in Husserl and Heidegger (2013). The book includes then an afterword with Crowell’s with his comments and replies.

For this review, I focus on the direct objections to Crowell’s philosophical positions and his attempts at answering them. For doing so, I follow the order proposed by the editors of the book. The book is divided into five sections: (1) “Normativity, Meaning and the Limits of Phenomenology”, (2) “Sources of Normativity”, (3) “Normativity and Nature”, (4) “Attuned Agency”, and (5) “Epistemic Normativity”. At the beginning of each section in this review, I offer a brief summary of the main ideas in each section of the book and then a brief commentary on each single chapter.

I. Normativity, Meaning, and the Limits of Phenomenology

This section is focused on the link between the question of normativity and that of meaning as it is addressed in phenomenology. Thus, normativity of meaning appears to be one of the main questions of phenomenology. However, several questions remain open which the following articles try to solve. First of all, the concept of norm can be understood in different ways and opens thus the question to the possibility of different normative structures for different meaningful experiences. This question is raised by Sara Heinämaa in her article which opens this section. A second question is raised by Leslie MacAvoy regarding the legitimacy of considering the structure of meaning as fundamentally normative, arguing that this would go against Husserl’s virulent critique of psychologism. She thus distinguishes the validity of meaning from its eventual manifestation for us as an ought or as a claim. The third one is raised by Zahavi, Cerbone, and Kavka. They challenge the idea in itself that the normativity of meaning is one of the main concerns of phenomenology. Thus, some realms such as metaphysics (Zahavi), epistemology (Cerbone), or philosophy of religion (Kavka) seem to be out of reach for the phenomenological method understood as a “metaphysically-neutral reflective analysis of the normative space of meaning” (Burch, Marsh & McMullin, 2).

  1. Constitutive, Prescriptive, Technical, or Ideal? On the Ambiguity of the Term “Norm”, Sara Heinämaa

The starting point of this article is the claim that all intentionality, from a phenomenological point of view, has a normative structure, because all intentionality can be fulfilled or disappointed. Thus, every intentional object is a norm that can be fulfilled or disappointed. Heinämaa calls this type of norm a “standard”. However, following Husserl’s distinction between interested perception and thing-appearances, she shows that the intentional object as norm can have a second meaning, which is an unachievable goal and thus also an optimum. Indeed, thing-appearances can never be fully given to us in all their richness.

This polysemy of the notion of norm leads Heinämaa to analyze its different meanings by drawing on the work of the logician and philosopher Georg Henrik von Wright Norm and Action: A Logical Enquiry (1963). Wright himself takes over a distinction which he finds in the works of Max Scheler and Nicolai Hartmann, namely that between norm as actuality, which is at stake for what Husserl calls practical intentionality, and norm as ideality, which is essential for axiological intentionality. This distinction corresponds to Scheler’s distinctions between “Tunsollen” and “Seinsollen” and between “normative ought” (normatives Sollen) and “ideal ought” (ideales Sollen). The normativity of doing, which is a “normative ought”, is based on the concept of rule-following while the normativity of being, which is a “ideal ought” is based on the concept of seeking to achieve something. Both types of normativity should be kept strictly distinguished. Thus, although both types of normativity are goal-oriented, ideal norms “are not motivational causes for our actions but are conditions that define ways of being” (Heinämaa, 20).

Heinämaa applies this distinction to the question of the normativity of intentionality by arguing, against Crowell, that both Heidegger and Husserl, share the idea that norms of actions but also of thinking are founded in ideal norms. Thus, one of the roles of phenomenology is “to illuminate the fundamental role that ideal principles of being have in both epistemic and practical normativity” (Heinämaa, 23-24).

Steven Crowell insists, however, on the fact that the concept of ideales Sollen is not a proper “ought” but a “should” in order to preserve the clear distinction between normative and theoretical disciplines.

2. The Space of Meaning, Phenomenology, and the Normative Turn, Leslie MacAvoy

The leading question of this article concerns the proper object of phenomenology: is it meaning or normativity? First, Leslie MacAvoy shows how phenomenology, in its concern with meaning, takes over the neo-Kantian question of validity (Geltung). The neo-Kantians understand the validity of a logical law in terms of normativity, contrary to Husserl and Heidegger, and this explains the concern of this article.

Husserl argues in the Logical Investigations that logical laws are not normative because they are not prescriptive, and consequently that they are not practical rules but theoretical laws. Although these laws have normative power for our thought, normativity is not part of their content. In that way, what is opposed to the law of nature is not, contrary to what neo-Kantians thought, a normative law, but an ideal law. Therefore, contrary to Neo-Kantianism, phenomenology distinguishes validity from normativity. According to a phenomenological criticism, “the phenomenological critique of the neo-Kantian notion of validity as normativity transforms the space of validity into a space of meaning” (MacAvoy, 41). What is thus at stake are not the laws that “hold” but the intelligible structures of content. According to MacAvoy those structures are a priori and it is due to them that sense or meaning presents to us as valid. Here MacAvoy refers to Heidegger’s theory of the fore-structures of meaning as a model to understand this a priori, but she concludes, nevertheless, that phenomenology should investigate the sense of this a priori with more depth. All in all, while MacAvoy agrees with Crowell’s claim that phenomenology opens us the “space of meaning”, against Crowell’s Normativity and Phenomenology in Husserl and Heidegger, she disagrees with the idea that this space is normative.

3. Mind, Meaning, and Metaphysics, Another Look, Dan Zahavi

Zahavi’s concern in this article is the role of metaphysics in Husserl’s transcendental (and not early) phenomenology. Is his transcendental phenomenology metaphysically committed or does the epoché on the contrary entail metaphysical neutrality? By developing his argument, Zahavi critically assesses Crowell’s claim that Husserl’s transcendental phenomenology is metaphysically neutral. Crowell’s argument is that phenomenology is not interested into metaphysics but into “understanding the sense of reality and objectivity” (Zahavi, 50).

To this argument Zahavi presents two counterarguments. First of all, the fact that phenomenology is not primarily interested into metaphysics does not entail the fact that it does not have metaphysically implications. Secondly, Zahavi puts forward texts of Husserl where he explicitly claims the metaphysical commitment of phenomenology. In the Cartesian Meditations Husserl states that “phenomenology indeed excludes every naïve metaphysics that operates with absurd things in themselves, but does not exclude metaphysics as such” (Husserl 1950, 38-39). In order to understand the meaning of this metaphysical commitment of phenomenology, Zahavi distinguishes between three definitions of metaphysics: (1) “a theoretical investigation of the fundamental building blocks, of the basic “stuff” of reality” (Zahavi, 51); (2) “a philosophical engagement with question of facticity, birth, death, fate immortality, the existence of God, etc.” (Zahavi, 52); (3) “a fundamental reflection on and concern with the status and being of reality. Is reality mind-dependent or not, and if yes, in what manner?” (ibid). Zahavi further argues, that it is the second and most of all the third definition of metaphysics that is of interest for Husserl’s phenomenology.

Zahavi’s argument, drawing on an argument presented already by Fink in an article from 1939, “The Problem of the Phenomenology of Edmund Husserl”, is that transcendental phenomenology does not investigate the structures and meaning of a mental realm, but of the “real world” and of its modes of givenness (Zahavi, 59). Similarly, Fink insists on the distinction between the psychological noema and the transcendental noema, which is “being itself” (Fink 1981, 117). The distinction between noema and the object itself is not valid anymore within the transcendental attitude, but makes sense only within the psychological one.

To Crowell’s argument that Husserl is not dealing with being or reality itself but with its meaning for us, Zahavi answers that transcendental phenomenology entails a metaphysical claim about the existence of consciousness. However, the question remains open regarding the metaphysical commitment to the existence of a being which is independent from our consciousness, and this question is raised for example in Quentin Meillassoux’ book After Finitude, in which the author claims that phenomenology is unable to think being itself, independent from its correlation to consciousness. Of course, one could argue that Husserl dismisses this question, which he identifies as the Kantian question about things in themselves, as being absurd. However, perhaps we should investigate more why this question is considered being absurd by Husserl: is it not precisely because, according to him, it makes no sense to consider a being without presupposing a consciousness for whom this being has a meaning? There is, according to me, something very intriguing about this argument, in that it cannot be classified neither as metaphysical, since it does not claim that being is ontologically dependent on our consciousness, nor as semantically epistemic, since it does not claim that there is something as a neutral being which is then given to us through meaning. It would be interesting to, first, identify what type of argument Husserl actually uses here in order to deepen the question regarding the metaphysical commitment of phenomenology.

Opposing Zahavi’s argument, Crowell maintains his position concerning the metaphysical neutrality of phenomenology, which is guaranteed, following him, by the distinction between the existence of some entities, which is mind-independent, and the access to their reality, which is possible only for a conscience. Accordingly, however, this distinction still leaves the question unanswered concerning the metaphysical status of this reality to which we have access, since it still does not say how far this reality, as we have access to it, is mind-dependent.

4. Ground, Background, and Rough Ground, Dreyfus, Wittgenstein, and Phenomenology, David R. Cerbone

The aim of this article is to challenge Dreyfus’ interpretation of Heidegger’s concept of background as the understanding of being. According to Dreyfus, there is something as a background for our understanding which is there and that we can reach. Or, Cerbone argues for a deflationary sense of background which entails that there is no something as an ultimate background for our understanding, but always a changing and indeterminate background that we can never reach as such. Every time we try to explicate this background we always remain in his space. Thus, this background has an “illusory depth” (Cerbone, 76), since we can never get at its bottom.

In order to argue this, the author is drawing on Wittgenstein’s concept of explanation of the Philosophical Investigations. According to Wittgenstein, there is no absolute explanation of the background of the understanding, for example of the meaning of a word, but it is always relative to one specific situation and to the specific knowledge of our interlocutor. In that sense, explanations respond to a specific question or problem. They end when they fulfill that purpose.

This reassessing of the concept of background opens according to Cerbone the possibility to reassess the idea of phenomenology as infinite task. Indeed, the infinite task of phenomenology is not that of explicating the background of every understanding but that of addressing “the ongoing ethical challenge of making sense of and to one another” (ibid).

Steven Crowell objects however that Cerbone’s argument “seems to conflate the transcendental project of clarifying meaning with the mundane project of explaining some meaning by making the background explicit” (Crowell, 336). Crowell further argues that this argument makes it impossible to determine what is the world, since it is a category. Categories however are not explicated by ““digging deeper” into some specific horizon … but by phenomenological reflection on the eidetic structure of being-in-the-world” (Crowell, 337).

5. Inauthentic Theologizing and Phenomenological Method, Martin Kavka

This article examines the possibility of an authentic phenomenology of religion, which would be based on the authentic thinking of God. Martin Kavka understands here the concept of authentic thinking in the Husserlian sense in the way it is presented in the Logical Investigations, i.e. as the fulfillment of claims made in statements through the intuition of states of affairs.

Drawing on Heidegger’s analysis of Husserl’s concept of categorial intuition, from his 1963 essay “My Way to Phenomenology”, Kavka comes to the conclusion that an authentic phenomenology of the ‘inapparent’ must be possible, since categorial intuition is the intuition of an inapparent, i.e. a senseless, being. However, Kavka does not consider that God could be the object of such a phenomenology, as it is for instance in the case of Marion’s phenomenology of revelation, since religious figures such as Jesus are not fully dissimilar from the horizon of human expectations. The criterion of the phenomenon of revelation according to Marion lies precisely in its radical heterogeneity “to any conceptual scheme and horizon” (Kavka, 93); and since we could argue here that Jesus cannot precisely be simply identified to God, the question of Marion’s revelations is left open to possibility.

Following the question which Heidegger inquires in On Being and Time, Kavka asks himself what is the ground of meaning, and implicitly, if this ground can be considered as being God. He argues, following Hannah Arendt, that in any case God cannot be considered as commanding to our consciousness since this would “not lead Dasein back to itself and its own-most potentiality-for-Being” (Kavka, 90). Indeed Dasein cannot be ruled by any predetermined norm but can only respond to the call of normativity by responding for norms and making them its own.

Finally Kavka endorses Crowell’s horizontal analysis of discourse[1] in order to explain the primacy of alterity in Levinas’ sense, suggesting perhaps that such an analysis could also be of use for an authentic phenomenology of God, but most of all, for a critical philosophy of religion.

Steven Crowell argues however that a theological phenomenology would not be a phenomenology anymore since it would go beyond the “askesis of transcendental phenomenology” (Crowell, 352) due to which phenomenological investigations cannot but remain the realm of the evidence. Ending on a Kantian note, Crowell writes: “We are finite creatures, and so meaning is finite. We can grasp the world as it is, though never as a whole; and if there is anything beyond that, it is a matter for faith, not philosophy” (ibid).

II. Sources of Normativity

This section explores the sources of normativity both from a phenomenological and from an analytical point of view. John Drummond argues, from a Husserlian perspective, that these sources lie in the teleological structure of intentionality, whereas Inga Römer highlights, from a Levinasian approach the role of the other. Finally, Irene McMullin is arguing for the plurality of these sources (first-, second-, and third-personal) highlighting an unexpected feature of normativity: gratitude.

  1. Intentionality and (Moral) Normativity, John Drummond

In this article John Drummond argues against Crowell’s Heideggerian approach of the sources of normativity as being pre-intentional, For John Drummond, the intentionality is a “’basic’ notion” (Drummond, 102) which can ground by itself normativity. First of all, against Crowell’s reading of Husserl according to which the pre-intentional flow of consciousness constitutes intentional acts, the author argues that this flow is also intentional but is structured by a type of intentionality which Husserl calls in On the Phenomenology of the Consciousness of Internal Time horizontal intentionality (Langsintentionalität), which has the specificity of not being oriented towards an object, contrary to the transverse intentionality (Querintentionalität). Thus, “intentionality … belongs primarily to mind ‘as a whole’” (Drummond, 105), whereby mind has first of all the meaning of a gerund: “mind is ‘minding’ things” (ibid).

Secondly, Drummond highlights the fact that mind pertains to a person, i.e. to a concrete social, historical, embodied subject, which is for him equivalent, just as for Crowell, to the transcendental ego. Thus normativity has to be understood as the telos of the intentional experiences of a personal subject, which is aiming to truthfulness. This truthfulness presupposes that the person is responsible for acting and leading his/her life in the light of this telos.

The author concludes that this telos governs our lives as individuals and communities. My question would be however: what allows the author to be so sure about the universality of this telos? Could we thus say that truthfulness is still the goal of a totalitarian society for instance?

Steven Crowell objects to Drummond’s argument that horizontal intentionality, although it belongs to the ground of reason, is not however “governed by a telos of reason” (Crowell, 338). He argues instead, along with Heidegger, that what clarifies intentionality is the categorial structure of “care”. Thus it is in this structure of care that normativity is ultimately grounded.

2. The Sources of Practical Normativity Reconsidered – With Kant and Levinas, Inga Römer

Just as Steven Crowell showed that there is a “line of continuity” (Römer, 120) between the phenomenology of Heidegger of Being and Time and the philosophy of Kant, Inga Römer argues in this article that there exists such a line of continuity between Levinas’ phenomenology and the thought of Kant.

First of all, she shows how Levinas’ reading of Kant evolves, from a very critical one (until the 1960s) to a positive one, especially regarding the second Critique, from the 1970s. Thus Levinas starts to consider Kantian philosophy of pure practical reason as a “philosophy of the sense beyond being, a sense that is essentially ethical” (Römer, 123). At the same time, Levinas transforms Kant’s idea of pure practical reason by anchoring pure practical reason in the desire for the infinite, by grounding the autonomy of the self in the ethically signifying call of the Other and finally by reinterpreting Kant’s idea of pure practical reason as an anarchic reason. This anarchic reason involves a tension between the claim of the Other and the claim of the third, and thus a “pure disturbance, confusion, restlessness, and refusal of synthesis.” (Römer, 125)

Secondly, Römer considers in details and criticizes Korsgaard’s and Crowell’s arguments for grounding ethics in a first-personal perspective, by arguing that Levinas’ perspective is more convincing because “it is impossible to generate ethical rationality within myself … without falling into a sort of ethical self-conceit” (Römer, 132) which would make us unable to feel obliged towards the Other. Perhaps it would have been interesting to develop this concept of “self-conceit” since it is essential for the author’s argument.

Thirdly, Römer shows how Levinas’ thought is closer to the argument of the second Critique than to that of the Groundwork for the Metaphysics of Morals, because it is also based on the idea that ethical rationality, as a mere fact, institutes my autonomy. Levinas and the later Kant thus agree on one essential point: “it is impossible to generate ethical rationality by starting with my very own freedom and then extending it towards others” (Römer, 134). An important distinction remains however between these two thinkers, according to the author: contrary to Kant, ethical significance remains, following Levinas, threatened by nihilism, especially nowadays.

Steven Crowell answers to Römer’s critique by arguing that Levinas’s thought encounters the same problem as that of Heidegger, namely that the identity of the addresser of the call remains enigmatic and leads to metaphysics. On the contrary, the concept of categorial answerability for reasons, does not require metaphysics.

3. Resoluteness and Gratitude for the Good, Irene McMullin

In this article Irene McMullin’s aim is to understand deeper the original Heideggerian concept of resoluteness, which allows the agent to overcome Angst in order to act in a norm-responsive way. More precisely, she studies the affective dimension of resoluteness by studying what Heidegger calls “readiness for anxiety”. One of the main claims of this article is that this readiness is not a merely negative experience, because it implies also gratitude, which is “an essential affective component of resoluteness” (McMullin, 137).

First of all, McMullin nuances Heidegger’s idea according to which there are mainly two sources of normativity: the public conventions of the das Man and our private norms. She argues indeed that there is a third normativity source, which are second-person claims. She, then, insists on the importance of readiness for anxiety, which she interprets as a latent state of anxiety through which the Dasein takes into account the plural sources of normativity. This readiness is an affect and not a project, since the world matters to me through it. Finally the author insists on the dimension of joy which is essential for this readiness, since I experience gratitude when I consider the possibility of losing everything (for example a child), but which has not yet realized itself. We experience, thus, gratitude for the meaning of our life, because precisely we become conscious, through readiness for anxiety, of the contingency of this meaning. Thus, “gratitude is the orientation that responds to grace – meaning a manifestation of goodness over which we have no power, but to which we find ourselves gratefully indebted” (McMullin, 150). I remain however with one pending question: is it still possible to experience this gratitude when all meaning is lost, when we do not experience anymore the world as “overflowing with meanings that we do not create or control”? (ibid). Or is the absolute loss of meaning a necessary possibility following from the characterization of the meaning of our lives as being precisely contingent? Steven Crowell deduces from McMullin’s argument the interesting idea that “the phenomenological focus on meaning prior to reason does not lead to nihilism, then, but to fröhliche Wissenschaft” (Crowell, 342).

III. Normativity and Nature

This section investigates the relationship between phenomenology and naturalism, reinterpreted respectively as the relationship between the “space of meaning” and the “space of causes”, according to the expression used by Steven Crowell in his work Husserl, Heidegger, and the Space of Meaning: Paths toward Transcendental Phenomenology. All authors of this section aim to bridge the gap between the natural and the normative realm whether by showing that there is no essential distinction between human and nonhuman animal “selves”, by arguing for a “relaxed naturalism” or by showing that human intentionality can be understood as a natural phenomenon.

  1. On Being a Human Self, Mark Okrent

Mark Okrent investigates in this article what constitutes the human self. He first examines the classical answers to this question, from Descartes to Kant, by showing finally that Kantian answer is problematic for two reasons: it is not able to explain why a human agent could have a specific reason to act; it has a restricted view on rationality, reducing it to its deductive aspect. Korsgaard’s concept of “practical identity” can offer a response to the second problem. One of the essential dimensions of this practical identity is the overcoming of a passive dimension that we share with other nonhuman agents, i.e. the goal of self-maintenance.[2] Thus, being a human agent entails overcoming the passive dimension that we share with nonhuman agents in order to become normative agents. However, according to Okrent, Korsgaard is not able to explain for which reasons one should adopt a certain practical identity.

Secondly, Okrent examines Heidegger’s idea according to which one does not represent oneself a certain practical identity in order to act according to it, since it could offer an answer to the problem mentioned above. However, this idea is unable, according to Okrent, to make clear how a certain identity is one’s own achievement. Crowell’s answer to this objection is that no practical identity is merely given to us, even when we are not in the mood of anxiety, but that we have on the contrary to strive constantly to achieve this identity. Thus, if animals respond instinctively to their identity, human beings inhabit an indefinite identity to whose norms they try to respond. However, as Okrent mentions it, recent animal studies have shown, that animal identities are not merely instinctive, but can evolve in function of environmental conditions.

Okrent attributes however another possible meaning to the concept of trying to achieve an identity which Crowell uses: it does not mean to “alter” it “in the direction of greater success”, but also to “justify” it with reasons (Okrent, 173). However, this interpretation is confronted with the aporia of the Wittgensteinian regress, which thus puts into question Crowell’s argument for the radical difference between human and animal identity as agents.

Steven Crowell responds to Okrent’s argument by arguing that it involves a deep pragmatic “reconstruction” of Heidegger’s text and that it would be thus more “elegant” to leave aside pragmatism (Crowell 344).

2. Normativity with a Human Face: Placing Intentional Norms and Intentional Agents Back in Nature, Glenda Satne and Bernardo Ainbinder

The aim of this article is to prolong McDowell’s attempt to replace norms in nature in order to avoid Sellar’s and Davidson’s separation of the space of causes from that of reasons, to which belongs, according to Sellar, intentionality. Crowell considers however that McDowell lacks the necessary phenomenological account of perception, in order to show that perception belongs to the space of norms, without being conceptual, and thus in order to achieve empiricism.

In order to achieve this project, Satne and Ainbinder argue that it is essential to place intentional agents in nature, even if Crowell denies the possibility of an account of rationality in natural terms. According to the authors, Husserlian genetic phenomenology can provide us with a method in order to describe this, because it can show how our normative capacities emerge from more basic capacities that we share with children and animals, and thus with other nature beings. They posit thus themselves against Crowell’s view according to which there is a radical gap between human intentionality, which is the proper intentionality and animal intentionality, or against Davidson’s view according to which we lack the proper vocabulary in order to describe the mental states of other animals. This is what allows them to give an evolutionary account for human intentionality.

In order to achieve this project, Satne and Ainbinder criticize what they call the uniformity thesis according to which intentionality is “the exclusive province of semantic content” (Satne & Ainbinder, 188). This requires showing how a phenomenological understanding of “life” allows to pluralize the “forms of life” and thus to pluralize intentionality. For this aim, the authors broaden the concept of nature so that it can include consciousness and so also intentionality. However, one question remains open: how is it possible, according to the authors’ projects, to reunite intentionality with the realm of nature understood in its mere biological sense, and thus with the neurological part which could correspond to intentionality?

Steven Crowell presents an objection to the argument presented in this article by advancing that it presupposes the use of genetic phenomenology and thus “a construction that transcends the kind of Evidenz to which transcendental phenomenology is committed” (Crowell 346-347).

3. World-Articulating Animals, Joseph Rouse

The aim of this article is to reunite, against Crowell’s and Heidegger’s views, our biological animality with our intentionality and normative accountability. Both Crowell and Heidegger insist on the incommensurability between animal environment and the openness to the world of the Dasein which creates a radical difference between animals and human beings. That is why it is not possible according to Crowell to ground normativity nor intentionality on the basis of “organismic teleology” (Rouse, 206). What allows us to attribute intelligibility to other animal forms of life is precisely the “transcendentally constituted space of meaning and reasons” (ibid).

In order to reject this argument, Rouse is arguing for a non-dualistic conception of normativity and nature. He thus proposes an “ecological-developmental conception of biological normativity” (Rouse, 207). which accounts for the development of normativity through social practices inside of which human beings grow up and live. These practices presuppose the essential interdependence of human being’s actions that is based on a mutual accountability of human being’s performances. Their normativity reside in this accountability and not in specific norms which would govern these practices.

This normativity without norms of social practices constitutes the specificity of human normativity, because it is two-dimensional: “whereas other organisms develop and evolve in ways whose only measure is whether life and lineage continue, our discursively articulated practices and their encompassing way of life introduce tradeoffs between whether they continue and what they ‘are’” (Rouse, 210). A question remains however unanswered: on the basis of which arguments can we be so sure that our normativity presupposes a biological dimension which urges us to continue life and is thus two-dimensional? What allows us to argue that the evolutionary development of our normativity did not on the contrary suppress this dimension?

Crowell’s reply to Rouse’ criticism is that he does not take the dualism between nature and normativity in a metaphysical but only in a methodological sense, since phenomenology is metaphysically neutral. Further, Crowell’s argues that we are led to deduce from Rouse’ account the problematic idea that phenomenological categories are contingent.

IV. Attuned Agency

This section investigates the affective dimension of normativity. The first article challenges the view that we are not responsible for our moods, while the second one nuances from a phenomenological point of view the traditional description of akrasia and its relationship to conscience. Finally, the third article investigates how normativity is intricate in the experience of erotic love.

  1. Moods as active, Joseph K. Schear

The aim of this article is to challenge the idea that moods are a mere expression of our passivity, by arguing that they are on the contrary “an expression of agency for which we are answerable” (Schear, 217). Here, Schear criticizes the classical interpretation of Heidegger’s concept of Befindlichkeit (as that of Dreyfus or Mulhall for example) as manifesting the “passive” dimension of our being-in-the world.

The objective of Schear is radical, since he does not simply try to show that we can act on our moods, but that the fact in itself of being in a mood is already an expression of our agency, and thus of our responsibility. First of all, the author elucidates the concept of being active as “being responsive to reasons” (Schear, 222). The fact that we can ask someone why he is in a certain mood displays already a piece of evidence for the fact that moods are active. We are thus expecting answerability for our moods.

The author distinguishes this answerability from moral responsibility. Answerability means here rather the possible “demand for intelligibility” (Schear, 225). Finally, this demand for intelligibility is not a demand for rationality, since what is at stake, is not asking for a reason which justifies the mood, but for “an account that makes manifest, that expresses, the shape or tenor of one’s situation as it shows from one’s perspective” (Schear, 228).

The author seems however to presuppose that someone has enough self-knowledge in order to answer this demand for intelligibility. However, it can happen that someone does not know oneself why he/she feels in a special mood (this can be the case for example when someone suffers from depression or anxiety) or that he /she does not understand rightly what makes him /her feel in a special mood. I can thus think that I am anxious because of my work whereas what makes me actually anxious is a certain heavy perfume I wear. Consequently, this understanding would not be immediately obvious to me, but would require an exercise of critical self-reflection.

2. Against our Better Judgment, Matthew Burch

The scope of this article is to show that what is usually called akrasia, meaning the fact of acting against our own judgment, regroups actually two distinct phenomena that Burch describes from a phenomenological point of view. He thus defines the first phenomenon as “intention-shift: action taken freely and intentionally against my explicit plan (or future intention) and with a clear conscience” (Burch, 233) and the second phenomenon as akrasia in its proper sense, or more precisely: “action taken freely and intentionally against my explicit plan (or future intention) and accompanied by some self-critical emotion (e.g. guilt, shame, self-directed anger) or a mixture of such emotions” (ibid). The fundamental difference between these two phenomena lies in the negative, self-critical feeling that accompanies the second phenomena. Remarkably, both phenomena presuppose the free and intentional action, against the classical understanding of akrasia, which interprets it as a “conflict between rational judgment and irrational desire” (Burch 240). Burch shows on the contrary that what is at stake is a conflict between two interests, that he understands as being self-reflexive and normative. This conflict is understood by the author as a shift from a specific interest to another one, due to “affective and circumstantial changes” (Burch 242). According to the author, our interests are self-reflexive, because they concern ourselves. In the case of the intention-shift there is no betrayal of ourselves but only of our “prior plan” (Burch, 243) contrary to the akrasia in its proper sense. Thus, in this second case, shifting to another interest means also betraying another interest (e.g. being faithful to my partner), and thus betraying myself.

The author seems to presuppose that in the case of akrasia there is an asymmetry between two interests, which presupposes that satisfying one interest can lead to a feeling of self-betrayal (e.g. when I cheat on my partner), while this is not the case for the another interest (e.g. meeting other erotic partners than my wife/husband). Could we however think that this second type of interest can also lead to a feeling of self-betrayal when it is not satisfied?

3. Everyday Eros: Toward a Phenomenology of Erotic Inception, Jack Marsh

This article focuses on the phenomenological account of the earliest stage of erotic experience, that Marsh calls erotic inception. The author distinguishes several moments inside of erotic inception. The first moment is what he calls the standing-out-among, when the other catches suddenly our eye through a particular detail. The second moment is the stepping-out-from, when I step out toward this other who caught my eye. Through this second moment the other as potential erotic partner steps into my world. According to the author, this second moment is a modification of the Heideggerian concept of “world-entry” (Welteingang).

The author deepens then the understanding of this concept as applied to erotic inception, by deepening its Heideggerian description as upswing (Überschwingende). Marsh characterizes this upswing as an “ ‘oscillation’ between my possibilities and my facticity, my abilities and limits, my possible futures and actual past” (Marsh 260) and thus as an “excess of possibility” (ibid) or as “an expansive opening upon the world that is empowering and enriching” (Marsh, 261). This expansive opening upon the world leads finally to a world-modification that characterizes the unfolding couple. However, the excess of possibilities that characterize erotic inception contains also the possibility of its own demise, or as the author puts it, of the “We-death” (Marsh, 264).

One question remains however open for me: what place does the author attribute to normativity inside the erotic inception? Could we thus say that the experience of erotic inception is characterized by certain norms, like for example the norm of what it is to be an erotic partner, and that each of us can be called to transform these norms through one’s own experience?

V. Epistemic Normativity

This final section investigates the specific modality of normativity involved in our epistemic practices. The first article challenges the view itself that normativity is involved in knowledge acquisition, while the second article analyzes how norms are intricate in our perceptual experience. Finally, the last article investigates the link between the natural and the transcendental attitude from a phenomenological point of view.

  1. Normativity and Knowledge, Walter Hopp

In this article Walter Hopp deepens Crowell’s view according to which intentionality can be exercised only inside of a “context of practices” (Hopp, 271). Thus, “the world is not the intentional correlate of a transcendental ego, but the environment of the embodied and socialized human person” (ibid). Hopp argues that this idea could have two possible interpretations: either intentionality can be carried out only by persons who act conform to a context of practices and thus of norms, or intentionality is constitutively normative. Hopp is arguing for the first interpretation, by advancing that if intentionality were constitutively normative, then this would be the case for knowledge as well. He aims to show in this article that knowledge is not precisely constitutively normative, and so nor intentionality.

Hopp’s argument is based on Husserl’s theory of normative science from the Logical Investigations. A normative science according to Husserl is always based on one or several non-normative, theoretical sciences, like for instance medicine that is based on biology, chemistry, etc. Consequently, sciences that do no rest on other non-normative disciplines, like for example logic, cannot be normative. Non-normative scientific propositions can however endorse the role of norms, without being normative in their content. Hopp applies this argument to epistemology, by showing that its content does not indicate what we ought to believe but what can be hold as being true; or, truth can endorse the role of a norm but is not normative by its content. Here, Hopp specifies Crowell’s characterization of truth as a “normative notion” (Crowell 2013, 239) which is, according to him, ambiguous. The author is thus arguing clearly for a clear distinction between ethics and epistemology against Terence Cuneo for example.[3]

Nevertheless, Husserl defines noetics as the “theory of norms of knowledge” (Husserl 2008, 132) whereas evidence as self-givenness is characterized by him as the “ultimate norm … that lends sense to knowledge” (Husserl 1999, 45). Here however Hopp uses Husserl’s own criterion of normative science by asking on which non-normative discipline Husserl’s most fundamental concepts of his epistemology, i.e. truth and evidence do rest, in order to show that these concepts do not have normative content. This allows Hopp to define epistemology as an ideal science in the Husserlian sense.

Steven Crowell agrees with Hopp’s argument, but he points to the fact Husserl’s analysis of truth cannot be reduced to the Logical Investigations, which are essential for Hopp’s argument. According to Crowell, there is however a sense of normativity in Husserl’s transcendental phenomenology which “eludes Husserl’s distinction between normative and theoretical disciplines” (Crowell, 332) because transcendental phenomenology is not considered as an explanatory theory but as a method of clarification.

2. Appearance, Judgment, and Norms, Charles Siewert

The aim of this paper is to argue, by using the phenomenological approach, that our perceptual experiences are “subject to norms of its own” (Siewert 290). In order to show this, Siewert starts by analyzing the case of visual agnosia, by arguing that it does not involve a deficit of visual appearance but rather of capacity of recognition. Visual appearance is thus conceptually distinct from visual recognition, or recognitional appearance, which is on its turn distinct from judgment. Indeed, I can withhold judgment when I recognize two persons as looking the same, i.e. when I recognize that they look alike but in two different tokens. Visual recognitions “take thing as” (Siewert, 299) whereas judgments “represent things to be” (ibid). Contrary to Travis, Siewert does distinguish however altogether visual experience from accuracy, and thus does not attribute accuracy exclusively to judgment. Thus, I can accurately recognize a sign as an arrow, while it actually represents an alligator. In this case I made a “creative use of the appearance” (Siewert, 301). Siewert draws here a parallel with the Kantian scheme, since just as the scheme makes both theoretical judgment and aesthetic imagination possible, the recognitional appearance can support both a judgment and a creative use.

On the basis of this distinction between visual recognitional experience and judging experience, the author argues that these two types of experiences are governed by two different kinds of normativity. He agrees on this point with Susanna Siegel, but not on the specific form of normativity that characterizes visual recognition. Indeed, Siewert identifies visual recognition with a “looking-as-act” (Siewert, 303) which he understands as the active experience of looking, contrary to the “looking-as-appearance”, and which thus can be done well or badly, or which can be improved. Perceptual experiences can be thus subject to normative assessment because visual recognitions can be an activity.

3. Husserl’s and Heidegger’s Transcendental Projects, Dermot Moran

In this article Dermot Moran aims at understanding the meaning of phenomenology as transcendental philosophy. Inspired by Merleau-Ponty’s essay “The Philosopher and His Shadow”, he investigates how the transcendental and the natural attitude are intertwined and how the idea of such an intertwining relates to Husserl’s and Heidegger’s phenomenology.

Based on a very detailed studied of Husserl’s and Heidegger’s texts, Moran shows first how Husserl’s view on the natural and transcendental attitude evolves from the Ideas I until the Crisis, as well as how Heidegger criticizes the Husserlian concept of natural attitude, which according to him is a comportment (Verhalten) and not an attitude as such. At the same time, the author points to ambiguous points in Husserl’s thought, like the relationship between the natural attitude and naturalism, which leads to the reification of the world. Despite this ambiguity, Husserl is clear on the distinction between transcendental and natural attitude, which is relative to the first attitude as the only absolute attitude, because of its “self-awareness and self-grounding character” (Moran, 313). One can become aware of the natural attitude as such only through a “shift in the ego’s mode of inspectio sui” (Moran, 314) which is the transcendental reduction though which we can adopt the transcendental attitude. Thus one of the key roles of transcendental phenomenology is that of allowing us “to investigate attitudes” (ibid) such as the theoretical attitude which masks the original position of the transcendental subject.

Moran further reflects on the meaning of transcendental phenomenology with the aid of Merleau-Ponty’s reading of Husserl’s texts according to which the natural and the transcendental attitudes are deeply intertwined. This reading could explain why Husserl calls in the § 49 of the Ideas II the transcendental attitude as being natural.

Merleau-Ponty finds such an intertwining in Husserl’s idea of a passive pregiveness of the world which underlies all intentional acts and which is not the object of act intentionality but of what Husserl calls in Formal and Transcendental Logic fungierende Intentionalität and that Merleau-Ponty translates in the Phenomenology of Perception as operative intentionality (intentionnalité opérante), a concept which is equivalent according to Merleau-Ponty with the Heideggerian concept of transcendence.

Moran identifies this operative intentionality with what Husserl calls, also in § 94 of Formal and Transcendental Logic living intentionality. He further reflects on this concept of living intentionality, by arguing, based on a thorough study of Husserl’s texts, that the task of transcendental phenomenology is to aim towards a living not in the world but within the life of consciousness, which Moran interprets, following Husserl’s expression in Formal and Transcendental Logic as the realm of our internality (Innerlichkeit), a concept for which Moran discerns a Heideggerian resonance. Only transcendental reduction, and the transcendental attitude it leads to, can give us access to this internality, and not the natural reflection that is proper to the natural attitude. Thus, “the aim of transcendental phenomenology” is “to uncover this life of functioning consciousness underlying the natural attitude” (Crowell, 320).

In conclusion, this book allows us to have a renewed reading of one of the main problems of phenomenology, i.e. the problem of meaning. Particularly, the problem of meaning is treated in the light of the question of normativity. At the same time it links in multiple ways the phenomenological question of meaning with various contemporary compelling questions like that of naturalism. This makes this book particularly interesting. Yet, the question of meaning is unfortunately not always on the foreground, leaving sometimes the task of making the explicit link between the problem of meaning and the content of the articles to the reader. Perhaps however it is a mere consequence of the richness of its various perspectives on this topic.

Bibliography:

Crowell, Steven. 2002. “Authentic Thinking and Phenomenological Method.” In: The New Yearbook for Phenomenology and Phenomenological Philosophy 2: 23-37

Crowell, Steven. 2013. Normativity and Phenomenology in Husserl and Heidegger. Cambridge: Cambridge University Press

Cuneo, Terence. 2007. The Normative Web. Oxford: Oxford University Press.

Fink, Eugen. 1981. “The Problem of the Phenomenology of Edmund Husserl.” Translated by R.M. Harlan. In: Apriori and World: European Contributions to Husserlian Phenomenology, edited by W. McKenna, R.M. Harlan, and L.E. Winters, 21-55. The Hague: Martinus Nijhoff.

Husserl, Edmund. 1950. Cartesianische Meditationen und Pariser Vorträge. Edited by S. Strasser, Husserliana 1. The Hague: Martinus Nijhoff.

Husserl, Edmund. 2008. Introduction to Logic and Theory of Knowledge: Lectures 1906/07. Translated by Claire Ortiz Hill. Dordrecht: Springer.

Husserl, Edmund. 1999. The Idea of Phenomenology. Translated by Lee Hardy. Boston: Kluwer Academic Publishers.


[1] Steven Crowell, 2002.

[2] See Christine Korsgaard, The Sources of Normativity, New York: Cambridge University Press, 1996 and also Christine Korsgaard, Creating the Kingdom of Ends, New York: Cambridge University Press, 1996.

[3] Cuneo 2007. Cuneo is arguing that just as there are no “moral facts”, there are no “epistemic facts” either. (113)

Graham James McAleer: Erich Przywara and Postmodern Natural Law: A History of the Metaphysics of Morals, Notre Dame Press, 2019

Erich Przywara and Postmodern Natural Law: A History of the Metaphysics of Morals Book Cover Erich Przywara and Postmodern Natural Law: A History of the Metaphysics of Morals
Graham James McAleer
Notre Dame Press
2019
Paperback $35.00
148

Natalie Depraz, Anthony J. Steinbock (Eds.): Surprise: An Emotion?

Surprise: An Emotion? Book Cover Surprise: An Emotion?
Contributions To Phenomenology, Vol. 97
Natalie Depraz, Anthony J. Steinbock (Eds.)
Springer
2018
Hardback 88,39 €
X, 189

Reviewed by: Andrew Bevan (Centre for Research in Modern European Philosophy, Kingston University, London)

What is it to define an emotion? Or to categorise an experience as an emotion? This is the aim of this collection of essays, the result of a conference of 2013 with the same name that discussed ‘surprise’ and attempted to categorise it as emotion, feeling, affect or otherwise. The editors identify two main theoretical frameworks with which to approach the question: psychology and philosophy. They argue that, whereas psychology treats surprise as a primary emotion, philosophy relates surprise to passions which are then opposed to reason. With this split in place, they seek to question these frameworks: is surprise not also cognitive? Is it not embedded in language? And how is it to be related to personhood and the interpersonal and moral emotions? Already we see that the exercise of defining an experience as an emotion takes place within the traditional binaries of philosophical psychology: passion/reason, emotion/cognition, etc. Yet throughout this volume, perhaps the most surprising aspect of surprise is just how inadequate these traditional categories are and how the phenomenon under discussion will exceed and trouble these traditional binaries.

One immediate difficulty the volume is faced with is what to call that which is to be defined or categorized: what is this realm of undefined or uncategorised? What most general word can refer to it: ‘surprise’? At some level, all authors can speak to this uncategorised experience called ‘surprise’; there is some binding of word and experience such that all authors can write on its vagaries and varieties. Yet how is this to be disambiguated from similar terms like wonder, startle, glance, etc. as well as the translation of these terms from other languages, most notably that of wonder (thaumaston) which, as Plato argued in Theaetetus, ‘is the only beginning of philosophy’ (155d). This is the very problem the volume engages with and thus, in so doing can be read as continuing this long tradition of surprise as the beginning of philosophy.

Three main themes occur in all the authors’ discussions. The most commonly invoked criteria for surprise that all authors mention in some form or another is the frustration of expectations. For example, Steinbock delineates surprise not only as ‘an experience of unexpected givenness’ but as ‘the accommodation of us to the situation by being the acceptance of what I cannot accept’ (10). These expectations can be implicit or explicit and not merely cognitive: they are discussed through concepts like habit or bodily adaptation to an environment. It is then in the frustration of expectations, or the difference between expectation and actuality, that surprise arises. Authors use many concepts to characterise these expectations (dispositions, integrations, entanglements and habit) and their frustration (startle, rupture, punctuation, anxiety, novelty and reconfigurations).  But there is also room for concepts that convey a lack of surprise when expectations meet actuality (affinity, affordance).

The second commonality is the question of temporality: while most agree surprise involves a spontaneous, sudden, ‘rupture’ this is merely the first part of a temporal dynamic. Desmidt, for example argues ‘surprise is the structure of the temporal dynamic of emotional emergence’ (62).

The third point of agreement between most authors is that surprise is ambiguously valenced: surprise can be positive or negative and so appears to transcend any simple division into positive/negative valence.

But, whereas the authors tend to agree on these three main points, there is then much divergence in their characterisation of surprise. The main problem in comparing positions to agree any consensus and the possibility of answering the question of the volume is that the difference between the authors’ positions in part stems from different understandings of the terms being used to categorize ‘surprise’. For instance, if surprise is to be an emotion, there is little discussion or agreement of what an emotion is, nor its difference or identity to affect, passion, feeling etc. is. Some treat affect and emotion as synonymous, others as strictly different but few reflect on what they might mean nor what categorising surprise as one or the other would entail.

The authors who give most attention to this question are the two editors of the volume, Steinbock and Depraz and both invoke Kant to define emotions. Steinbock foregrounds Kant’s use of temporality to differentiate affect and passion: affect is sudden and rash in contrast to the duration of passions (12-13).  Steinbock then, despite the suddenness of surprise, argues surprise is part of a process of much longer duration. But he concludes not that surprise is a passion but that surprise ‘belongs to the sphere of emotions (and is not a mere affect)’ (13). Steinbock thus seems to equate passion with emotion. Furthermore, whereas affects are ‘feeling-states and pertain to who we are as psychophysical beings, where we would find experiences like pleasure or pain, being ill at ease, tickling and arousal,’ emotions – such as ‘regret, remorse, fear, longing and surprise’ (14-15) – are emotions because ‘they can occur without any essential relation to personal ‘otherness’ in that experience’. But ‘genuine’ emotions are those which ‘presuppose an “order” or even “disorder” of the heart – to use a phrase from Pascal – and are lived in some way toward some other as bearer of value in a ‘creative’ or personal manner’ (15). Here we see that the divisions of psycho-physical to ‘personhood’ are played out to differentiate affect from emotion.

Meanwhile, Depraz argues that in psychology, surprise is treated as an emotion. She again cites Kant but, unlike Steinbock, identifies emotion with affect (‘emotion, here as Affekt’, 26). This identification of emotion with the German Affekt has a psychological precedence perhaps beginning with William James in his Principles of Psychology. For Depraz, surprise ‘is not an emotion in the sense of a basic feeling like fear, anger, disgust, jor or sadness.’ Her main argument is that ‘surprise involves an emotional and cognitive component but results in a more encompassing and integrative circular (time, bodily, expressive-descriptive) phenomena’ (39). Depraz then invokes the concept of valence to undermine the idea that surprise is an emotion: valence characterizes more precisely the ‘affective dynamic of the surprise rather than emotion as such, which always remains a partial and static state’ (40). Although surprise is linked to emotional valence when associated with these emotions, it may also appear as ‘a neutral, mixed or epistemic emotion, i.e. as a violated expectation that affects both action and cognitive processing.’ (39).

Other authors tend to reflect less on the problem, focusing their attention purely on emotion (Desmidt, Brizard) or tending to identify emotion with affect (Livet mentions ‘affective attitudes’ (109), ‘affects or affective bursts (111), ‘emotional or affective attitude’ (112)). Although Brizard does state that startle, that can be used to assess emotional reactivity which can be ‘modulated by affective states’ (78). Sheets-Johnstone in insisting the body is not ahistorical or living, speaks of ‘affective dynamics that move through bodies and move them to move’ (83). Yet, quoting Jung, she seems to elide any difference between affect and emotion (85).  Emotions/affects are then qualitatively different: they have their own ‘distinctive qualitative kinetic dynamics’ (85).

At least three different approaches can be identified then: affect equals emotions; emotion is a type of affect; or affects and emotions are different. A fourth approach, however, is to avoid the whole problem by mentioning neither affect nor emotion – such is Casey’s singular approach: he instead likens surprise to glance, something that is perhaps less contentious and more familiar.

This difference in understanding and use of terms then makes the guiding question ‘Surprise: An Emotion?’ difficult to answer: it of course depends on what an emotion is. So when Steinbock argues surprise is an emotion, and Depraz that it is not, they are working with slightly different understandings of what emotion is. For Depraz, emotion is an affect, for Steinbock it is not. Yet both agree that the aspect that differentiates surprise as one or the other is temporality: surprise is not sudden but part of a more involved process.

Perhaps some attention to the terms being used (affect, passion, feeling, emotion) might yield a more productive discussion. The terms affect and passion in particular have a long and rich philosophical heritage and perhaps most significantly enter the philosophical discourse through its use by Cicero, Augustine and others to translate the Greek pathos. Now, whilst passiones is a transliteration of the Greek pathos with similar meanings, affectio already existed in Latin and is comprised of the prefix ad- + facio. Ad- usually adds a movement to or against something whilst facio has a very broad signification including to make, build, construct or produce. Passiones is also the root of our passive and thus this choice of translation would foreground an essential passivity to this realm of experience. Whereas, with the choice of affect, which can be active or passive voiced (‘to affect or be affected’ will become central to interpretations of Spinoza), it is the binary of active/passive that is paramount in discussions of Greek pathos.

However, Cicero, in Tusculan Disputations, chooses neither affect nor passion but uses perturbatio to translate πάθος. He prefers this to morbus, meaning ‘diseases’ because the Greeks also used πάθος for exaltation and joy which we cannot consider disease. Thus, already we see the problem of valence when it comes to choosing a term to characterize these experiences – the term itself cannot be valenced. Furthermore, by choosing pertubatio, Cicero makes a philosophical intervention in the reception of Greek philosophy by replacing medical metaphors with metaphors of movement and reintroducing into Latin a model of mind in Plato and Pythagoras who divided the soul into two: one of peacefulness that shares in reason and another that doesn’t, the seat of stormy emotions, motus turbidosPerturbatio captures this metaphorical domain as it is comprised of the prefix per- meaning ‘thoroughly, to completion’ and turbāre from turbo ‘to disturb’ and implicitly contains a sense of a passive initiation of something that must run its course which means that, for Cicero, it becomes imperative to avoid perturbations in the first place as once initiated they cannot be stopped but must flow to completion.

On a purely etymological level, this understanding of perturbation resembles that of emotion which derives from the Latin ēmovēre to move out, drive away or banish, for example, pain. In this choice of concepts it is an implicit negatively valenced motion (as turbo or moveo) that is foregrounded . However, from a wider perspective than mere etymology, Thomas Dixon’s From Passions to Emotions claims that by 1850, the category of emotion had subsumed ‘passions,’ ‘affections’ and ‘sentiments’ in most English-language psychological theorists such as Hume’s Treatise of Human Nature (1739-40). The increase in popularity of emotion arose from the 17th century consolidation of philosophies of individualism as well as a secularisation that sought to avoid the associations of passion and affection with the biblical and theological preferring emotion for its alternative network of relations to psychology, law, observation, evolution, etc. This resulted in differing causal explanations for the phenomena: whilst Christian philosophers assumed passions were the soul acting on the body, emotions then became the brain acting on the body. The scientific brain replaces the theological soul as agent.

This analysis of concepts reveals at root two alternative approaches adequately described by affect/passion and emotion. Whether separated or identified, however, they nevertheless share an implicit foundation in activity and passivity and in the metaphorical domains of theology, medicine and physics. The question as to whether surprise is an emotion, affect or other is therefore not philosophically, historically or politically neutral. And this question continues to haunt the pages of this volume: for the question of valence appears regularly as well as the question of active/passive. And the metaphorical domain continues to oscillate between a philosophical approach (mainly that of phenomenology) and a more scientific one of psychology and linguistics. Indeed, the sheer diversity of disciplines included in this volume (without any one dominating) – medical (depression), philosophy (phenomenology), science (psychology), theological (in the discussion of Paul) or language and literature – continues the question over which metaphorical domain to place the concepts in. Such a complex and multi-faceted problem does indeed touch on everything from language, linguistics, phenomenology, science and theology and it is therefore refreshing that this volume features accounts from all these differing approaches.

Moreover, the volume is enhanced through combinations of these disciplines: the introduction states the multidisciplinary approaches as ‘philosophy, psychophysiology, psychiatry and linguistics’ (vi) and mention early attempts at the interface of philosophy and linguistics, phenomenology and psycho-neuro-physiology or philosophy-phenomenology. Phenomenology, neuroscience, physiology, is an interesting and productive binding.

If this short history of the concepts used to describe this realm of experience reveals anything, it is perhaps how implicated in past metaphysics this whole discourse is. Thus, it might be productive to uncover how implicitly the authors depend on such a past metaphysics (notably that of a past metaphysics of coupled opposites derived from Greek philosophy) in approaching the central question posed by the book. Furthermore, perhaps the value of this book lies in its manifestation of a tension relating to how surprise appears to depend on and yet transcend these categories and conceptual histories of philosophy.

Sheets-Johnstone speaks directly to this question of past metaphysics when she complains of a ‘metaphysics of absence’ that leads to an ‘absence of the body below the neck’ (84). The traditional body/mind division is that which leads to this critique. But the influence of a past metaphysics of coupled opposites is felt most concretely with the numerous oppositions that continue to structure the problem field: positive/negative, approach/avoidance, and sympathetic/parasympathetic nervous systems not to mention emotion/cognition and emotion/volition. Such a metaphysics enables the very analysis Livet proposes in his concluding paragraph where he walks through eight possible combinations based on oppositional pairings of explicit/implicit, emotion/volition and the transition between the two. This then requires also that emotion be opposed to cognition and the whole realm of complexity is perhaps reduced to slotting aspects into a neat, three dimensional grid of implicit/explicit, emotion/volition, affect/cognition.

But perhaps the main oppositional pair that governs all these other pairs is the active/passive which features prominently in many authors’ discussions and may stem from the translation of the Greek pathos into a discourse of passivity. For example, Steinbock asks whether surprise is active or passive given that startle must be passive (10). For Livet, the active/passive is applied to the difference between emotions (passive) and volitions (active) and Livet argues both can actually be active either in an explicit, conscious or implicit way. But ultimately, Livet and Steinbock both demonstrate just how futile and inadequate conceiving something like emotions as passive or active is. Steinbock notes that the active/passive cannot adequately be applied to surprise for it cannot be purely passive but indicates transition from a more passive to a more active awakening (12). Often what is passive is said to be also active leading to them being active and passive at the same time and the whole point of the distinction to disappear.

The centrality of the active/passive together with the alternate history of mostly disturbing movements gives rise to a conception of affects as quantitative flows and is evidenced in the repeated mentions of intensity and valence. For example, in Depraz’s brief history of the concept of valence that began with Kurt Lewin in 1935. He proposed valence as a double-opposed movement of attraction and repulsion in reference to his force-field analysis of social situations. It defines the intrinsic attractiveness of an event, object or situation and, by extension, also the attractiveness of the emotion itself. This concept then became ‘an operative concept to define the very structural dynamics of emotions in psychology’ (41). Perhaps we could say more generally it is a metaphysics of coupled opposites that defines the structural dynamics of emotion implicit to psychology?

Given the privileging of their disturbing character, passions, affects or emotions are then treated as (or have to be differentiated from) external impositions disrupting purely self-present subjects that produces philosophies of defence that privilege sameness over difference. This approach would then consider surprise as negative or, at least, somewhat out of our control.

Furthermore, if surprise is based very much on this difference between world and self, the question of what is surprising – prominent mostly in the linguistics section – is problematised as it will vary from individual to individual. Philosophies might then seek to ‘master’ affects: because one could not know in advance whether a surprise would be negative or positive, it is better to resist them all together. This question of individuality presents a challenge to those papers that try to elicit surprise in experimental settings. Can surprise be identified in the absence of the experiencer and their expectations that are often implicit? This is perhaps why Steinbock differentiates surprise from startle – one could agree we could all be startled by a loud interruption but whether one is surprised by some of the examples might depend on one’s experience in the world, particularly in the case of police interactions (9). Perhaps this question underlies the difficulty inherent to the project of deciding whether surprise is an emotion.

Bloechl is perhaps most explicit in addressing this question. He writes that, if surprise depends on some difference between a subject’s expectation and actuality, ‘the intelligibility of the experience depends in some important measure on the condition of the subject and its relation to the world in which it lives’. He thus argues we can differentiate among surprises by attending to the context in which they occur (historical, cultural, personal-psychological, etc.). But, he adds, ‘without surrendering the possibility of grasping their inner unity in some irreducible essence (eidos)’ (119). What is it that remains the same across all differences in surprise, different expectations, different subjectivities? The experience of difference?

An important point to mention on this question of individuality and whether emotions like surprise can be said to be universal is the focus Ekman’s paradigm of ‘basic emotions’ based on facial expressions receives in Depraz, Brizard, Goutéraux, Celle et al. Although Ekman receives criticism in Sheets-Johnstone for ‘“the absence of the body below the neck”’ (84), his paradigm as a whole continues to pervade the psychological discourse of emotions despite major methodological criticisms coming from within and without psychology. Ekman’s paradigm has been coherently critiqued, particularly over its claims to cross-cultural comparison, most notably by Ruth Leys in her The Ascent of Affect.

So is there an alternative to this approach to affects and to surprise? Could we uncover such an alternative, manifest them in the same way surprise acts to manifest a difference between implicit expectations and actuality? Can a focus on surprise yield the very surprises needed to reveal implicit foundations? Perhaps surprise best offers such a path with its ambiguous valence problematizes any neat ascription to either positive or negative. Furthermore, whilst we may know surprise in itself, the details of its surprise is unique to each occurrence. And, in the surprise, we can learn the difference between our habitual, implicit being as it becomes manifested in the difference to the actual. Thus, affects here become a potential for individual growth and becoming rather than something to be defended against whilst retaining some universality for comparison and intersubjective understanding.

One such alternative is being drawn out by the work of Depraz for instance in her rejection of opposites for circularity (39). She argues, refreshingly, that ‘integrated emotions [like love, submission, etc.] show that we have to deal here with a three-dimensional dynamic model and not with a linear list of emotions opposed one to the other’ (29). She notes how phenomenology is uniquely positioned to enable such a synthetic integration of of cognitive, physiological, evolutionary and other aspects and her proposal is for a cardiophenomenology that places the emphasis not on the brain but on the heart partly because the heart-system is an integrative system and better recognizes the ‘unique dynamic circular living rythmic of such a system’ (48). The heart self-organizes ‘as soon as the embryo develops spontaneous contractions independently of the brain’ and integrates the nervous and brain system as well as performing a control function (48-49). The heart is both physio-organic and uniquely lived. You can’t feel your neurons but you can feel your heart and thus is ‘self-feelable’, an auto-affection. Thus the heart becomes, ‘the matrix of the person as both lived (affection) and organic (muscle), or again, the core of the weaving between the first- and the third-person experience of the subject’ (48).

Such an approach allows for physiological measures to get third-person perspectives on surprise as startle yet also allows for comparison with first-person perspectives on the feeling of those physiological measures. It also allows the experiential aspect not just a theoretical-textual approach so that individual differences in singular surprising events can be acknowledged. Surprise is thus the core-experience of a heart-centred, cardiophenomenology for Depraz.

This focus on the heart and its rhythmicity gives a more interactive circular dynamic than the perhaps active/passive transmissions of the brain from input to muscular output.  Instead of causal, sequential flows of neuronal pathways, of flowing out of movements that must be expended, which always eventually leads to the active and passive (the brain as active sending out of passive sensations or movements), Depraz enables a focus on integration and circularity.

Desmidt also mentions cardiac psychology as ‘an integrative dynamic that includes the systems of the context, the body (and the heart and brain within the body), and the lived experience that dynamically interact according to the three phases to produce an emotional experience’ (64). He quotes Craig’s model of emotion in which an emotional experience ‘is produced by the sequential integration in the insular cortex of five types of information according to a spatial gradient’ (66).

Yet is this a move that repeats the debate between Galen and Aristotle – Aristotle seeing the heart as the centre, Galen the brain? For the nervous system is also seen as integrative. Perhaps the ultimate issue here is not whether it is the brain or the heart that is central but the challenge to the dominance of the active/passive ‘sending out’ for one that is more about circular dynamics.

Livet also acknowledges there should be a focus on ‘the entanglements between the different aspects of motivation experience […] without taking for granted restrictive definitions that overestimate their oppositions and underestimate their intimate relations. He urges a study of the ‘entanglements between different aspects of motivational experience without taking for granted restrictive definitions that overestimate their oppositions and underestimate their intimate relations’ (114). As to the active/passive, Livet recognizes that emotions are usually considered passive whilst volitions active but proposes they be considered as two kinds ‘that belong to a more inclusive category, namely the category of motivational dynamics’ (105).

It is then a question, not of oppositions but of entanglements, bindings, integrations that cannot be reduced to couplings of opposites or mechanical linear flows of active and passive but instead opens to the question of what bindings might enable and sustain our flourishing. Bloechl can perhaps be read as providing how an affect such as surprise could lead to our becoming and not be something to be defended against, mastered or known in advance through the example of Paul. Notably, Bloechl attends both to Paul’s state such that he experienced the surprise of a conversion (which depended on Paul’s ‘disposition’) as well as how he then integrated the experience. He looks for evidence for the former by attending to Paul’s Judaism prior to the experience and the latter through the Christianity in Paul’s letters.

What Bloechl concludes is that in Paul’s experience, and perhaps the experience of surprise more generally, there is a passage from inward and personal experience to an outward and universal discourse. He adds, ‘unless there is an affinity […] between that which surprises and that which is interpreted as the surprise, the event itself is literally unintelligible’ (127-128). This ‘affinity’ could also be called a context and it is surprise which can alter the entirety of a context, it comes, he adds, with ‘its own horizon of meaning’. Yet ‘unless at least some of this new meaning can be fused with the meaning of what it may challenge and transform, it remains strictly alien. The nature and limits of that fusion are open to interpretation and call for concepts that do not obscure the experience in question’ (128). Surprise and affects not so individual as to be incomparable across individuals or cultures but not so universal as to preclude the first-person perspective. Somewhere between reductive binaries and trivializing infinities.

Such an individual/universal approach is demonstrated in the volume applied to depression which is conceptualized in terms of an inability to anticipate pleasure in a situation even when they do then feel pleasure in its actualisation. Yet, it is a pity this account did not take into account the individual histories of expectation/actuality that is so paramount to surprise – if someone is depressed and cannot anticipate pleasure in a situation perhaps it is because of so many failed expectations? Although the authors suggest ‘hyporeactivity in depression may be characterized by an imparied cardiac physiology, especially during the anticipation phase’ (67). Here the question of individual history and ahistorical biology rears its head and the benefit should surely be in their mutual cooperation.

Perhaps if there is one key theme emerging from all these discussions it is the question of difference; difference between emotion and cognition, a difference encountered in an organism’s interaction with itself and its world that leads to differentiations, splits, retreats or avoidance and it extending or protending itself into its past/future. This focus on difference also helps against one discipline dominating: where is the organism’s self-difference? In the neurons? The gap between neurons? Any criticism of a cognitive privilege could then be countered by the fact that these expectations are often implicit and, moreover, manifested in the difference experienced and thus prior to any split between mind and body, this split coming after the fact as an attempt to integrate the experience. Indeed, it could be through a historical series of surprises that we find ourselves in this problem of mind/body dualism split. Is the feeling of oneself then arising from a unity with oneself or difference to oneself?

There are several mentions of the entangled nature of emotions and surprise. Can these be best understood within a metaphysics of opposites such as of active/passive, of cause/effect any longer? Or is the domain emotions try to capture one more of contingency, of expectations meeting actuality where these are not opposites but in their unfolding produce each other. Just like Picasso’s quote ‘je ne cherche pas, je trouve’ cited in this volume: it is only in finding, in the difference between expectation and actuality, that one knows one was searching.

It is in the unfolding of the entanglement this collection of essays resides in rather than the entanglement itself where surprise and emotion surely lie. Otherwise, we cannot truly find the alternative to the dominance of cognitive and computational so many authors descry. It seems if universality is not acceptable, and definitions vary, the experience of defining affects is the very experience of individuating, growing and self-differentiation, this self-differentiation that is the universal. Is this not a more adequate account of the affect surprise? Such would be the performative and not merely textual effect of reading this volume. Today, perhaps it is not wonder but surprise that is the beginning of philosophy.

Martin Kusch (Ed.): The Routledge Handbook of Philosophy of Relativism, Routledge, 2019

The Routledge Handbook of Philosophy of Relativism Book Cover The Routledge Handbook of Philosophy of Relativism
Routledge Handbooks in Philosophy
Martin Kusch (Ed.)
Routledge
2019
Hardback £175.00
600

Frode Kjosavik, Camilla Serck-Hanssen (Eds.): Metametaphysics and the Sciences: Historical and Philosophical Perspectives, Routledge, 2019

Metametaphysics and the Sciences: Historical and Philosophical Perspectives Book Cover Metametaphysics and the Sciences: Historical and Philosophical Perspectives
Routledge Studies in Metaphysics
Frode Kjosavik, Camilla Serck-Hanssen (Eds.)
Routledge
2019
Hardback £115.00
292

Matthew Burch, Jack Marsh, Irene McMullin (Eds.): Normativity, Meaning, and the Promise of Phenomenology

Normativity, Meaning, and the Promise of Phenomenology Book Cover Normativity, Meaning, and the Promise of Phenomenology
Routledge Research in Phenomenology
Matthew Burch, Jack Marsh, Irene McMullin (Eds.)
Routledge
2019
Hardback £120.00
358

Reviewed by: Steven DeLay (Old Member, Christ Church, University of Oxford)

On one common telling of the history, phenomenology originates as a philosophical movement incubated in professional jealousy, personal rivalry, and intrigue. If someone as Emmanuel Falque has called the recent work among phenomenologists in France a “loving struggle,” the same cannot be said for phenomenology’s earliest beginnings in Germany. Surrounded initially by a burgeoning cadre of students whom he hoped would be heirs to a research program united in its philosophical vision, Edmund Husserl, father of transcendental phenomenology, instead found his aspirations increasingly disappointed as the years passed. As he was to remark in a note towards the end of his career, the general sentiment of his time, one against which he never ceased to struggle, took a dismissively dim view of the systemiticity he so favored: “Philosophy as science, as serious, rigorous, indeed apodictically rigorous science—the dream is over” (Husserl 1970, 389). At the end of his life, he stood alone in his unflagging zeal for the cause of philosophy as science. One after another, Husserl’s former disciples with rare exception had deserted that vision of phenomenology and its future. Among the most notable of those to go their own way rather than following Husserl’s was Heidegger of course, who, beginning with 1927’s publication of Being and Time, broke publically with his mentor’s view of philosophy as a rigorous science, abandoning phenomenology as a science of trancendental consciousness for fundamental ontology’s Seinsfrage.

Expansive and sometimes rather convoluted, the details of this acriminous yet vibrant phenomenological milieu’s institutional reception (first across Europe then on to the Anglophone world and beyond) is far too complex to summarize here fully. Entire books have been written on such matters. Nevertheless, it is worth noting that there has for many decades existed a tendency on the part of commentators to reinforce the feud between Husserl and Heidegger. Rather than looking for any deep common ground between their philosophies, focus instead has been payed to highlighting the differences thought to separate them. This is particularly true in the North American context. For instance, when Hubert Dreyfus upon developing his criticisms of Artificial Intelligence at MIT brought Heidegger’s philosophy to students at Berkeley (William Blattner, Taylor Carman, John Haugeland, Sean Kelly, Iain Thomson, and Mark Wrathall among them), his presentation of phenomenology, which became a commonplace in many publishing circles, relegated Husserl to a piñata for Heidegger. From the 1970s on, Dreyfus’s reading dominated considerable portions of the Anglophone phenomenology world as orthodoxy. The picture it presented was tidy. Husserl was the antiquated cartesian who had underestimated the importance of matters like embodiment and intersubjectivity, while Heidegger and Merleau-Ponty, representatives of a so-called “existential phenomenology,” were pioneers whose innovative emphasis on being-in-the-world freed phenomenology from the history of philosophy’s misleading assumptions. In the rush to accentuate what it believed makes Heidegger’s philosophy captivating, Husserl unfairly became something of a footnote to the story, a sort of hors d’oeuvre before the main philosophical dish.

A notable exception to this trend is Dan Zahavi, whose work has done bright things to vindicate the continued importance of Husserl’s legacy. But perhaps the one who above all is responsible for snatching Husserl from the jaws of misunderstanding is Steven G. Crowell, who, in books as Husserl, Heidegger, and the Space of Meaning: Paths towards Transcendental Phenomenology (Northwestern: 2001) and Normativity and Phenomenology in Husserl and Heidegger (Cambridge: 2013) as well as in numerous essays has developed an iconoclastic and sophisticated account of the relation between Husserl and Heidegger. Crowell’s position is one which maintains, against Dreyfus and much of the received wisdom in Anglophone Heidegger studies, that in fact Husserl and Heidegger are collaborators in the shared undertaking of what Crowell himself characterizes as transcendental phenomenology’s distinctive project: namely, its preoccupation with the normative structure of intentional meaning (Sinn). Thus, at stake in the collection of essays contained in Normativity, Meaning, and the Promise of Phenomenology is the very status of phenomenological philosophy as Crowell proposes it be understood, as a transcendental “clarification of meaning” (Crowell, 336). Naturally, continual reference throughout is made to the interface between Husserl and Heidegger, but not, note well, for the purposes of mere exegesis, but instead as a wellspring of inspiration for a philosophical legacy whose unique approach to phenomenology is animating the continued work of thinkers carrying on its tradition. Many of the essays are accordingly not the typical kind of banal laudatory pieces one is accustomed to finding in a Festschrift. For, in paying homage to Crowell’s vision of transcendental phenomenology, they aim to return to the “things themselves,” precisely as Crowell himself has for many years urged others to do. In short, this is an excellent volume whose aim is not so much to read Husserl and Heidegger, but to think with, and, where necessary, against them.

This transcendental approach—or, a “critique of meaning”—is exemplified in Crowell’s own contribution to the volume. In an “Afterword” that closes the discussion by answering the essays preceding it, Crowell begins his response by noting how the grand language Husserl himself frequently employed when trying to convey the discovery of transcendental phenomenology’s significance may lead to some puzzlement. As Crowell recognizes, Husserl’s personal enthusiasm at first could seem a touch overstated.

With his turn to transcendental phenomenology, Husserl increasingly spoke of his work in the most exalted terms. He was Moses taking the first tentative steps toward the “promised land” whose riches he would not exhaust had he the years allotted Methuselah (Husserl 1989, 429); he was the explorer of “the trackless wilderness of a new continent” (1989, 422) where “no meaningful question” is left “unanswered” (Husserl 1970a, 168); he was Saul on the way to Damascus, the discovery of phenomenology affecting him like a “religious conversion” (Husserl 1970a, 137); he was the redeemer of “the secret yearning of all modern philosophy” (Husserl 1983, 142). What could motivate such language? (Crowell, 329).

According to Crowell, Husserl’s exuberance becomes understandable when the latter’s fundamental philosophical insight is appreciated properly. Husserl’s phenomenological breakthrough, says Crowell, lies not so much in the thesis that “intentionality is the mark of the mental” (as Franz Brentano had noted already), but rather in its distinctive concern with (to borrow the Heideggerian phrase) a kind of “ontological difference”: philosophy is seen to thematize not entities, but meaning. Further, the focus is not just on meaning but specifically the fact that such meaning is normative: “Phenomenology’s promise land, meaning, has a normative structure” (Crowell, 330). Hence, for Crowell, modern philosophy’s transcendental turn (as represented by Husserlian phenomenology) is at once a “normative turn” (MacAvoy, 29). It is in this context that the phenomenological reduction should be understood.

“[T]his method,” says Crowell, “requires askesis, suspending worldly commitments. I ‘put out of action the general positing which belongs to the essence of the natural attitude’ and ‘make no use’ of any science that depends on it (Husserl 1983, 61) so as to thematize the inconspicuous phenomenon of meaning, where the world and everything in it is available to us as it in truth is. This askesis characterizes all phenomenological philosophy” (Crowell, 329).

With this “reduction” to meaning, a new field of inquiry opens, one Husserl in works like Cartesian Meditations characterizes as “an infinite realm of being of a new kind, as the sphere of a new kind of experience: transcendental experience” (Husserl 1973, 66). And as Crowell contends, it is this reduction to meaning that unifies those thinkers belonging to the tradition of transcendental phenomenology. Moreover, it is the normative approach’s distinctive clarification of meaning that holds out the promise for re-establishing today the kind of research program Husserl had sought for his own. An approach calling for collaborative effort, not only does it promote the open exchange of ideas through critical argument, it does so while always remaining oriented by a methodological commitment to phenomenological Evidenz, the distinctive warrant of what shows itself intuitively in first-person self-givenness.

Husserl insists that phenomenology is not a “system” deriving from the head of a single “genius” (Husserl 1965, 75), but a communal practice, a “research program” in the loose sense that analytic philosophy might be considered one. What unites this program—including Heidegger, Sartre, Beauvoir, Merleau-Ponty, Levinas, and even Derrida—is a “reduction” from our ordinary concern with entities, beings, the “world,” to the meaning at issue in such concern. Of course, these and other practitioners interrogate both the reduction and the meaning it brings into view, and so we who take up the promise of phenomenology must assess, by our own lights, the legitimacy of such “heresies,” revisions, and revolutions. And while criticism of arguments is always in place, assessing the legitimacy of phenomenological claims finally requires Evidenz, what one can warrant for oneself in the intuitive self-givenness of the “things themselves.” As a kind of empiricism, phenomenology embraces the responsibility of first-person experience (Crowell, 330-31).

However, if transcendental phenomenology’s concern is meaning, and such meaning in turn concerns the normative, what is a norm? As Crowell recounts, a form of that question has long vexed philosophy’s effort to comprehend the realm of “ideality”: it led Plato to his theory of Forms, just as it later motivated nineteenth-century thinkers including Emil Lask and Hermann Lotze to their respective accounts of Geltung (validity), of a “third realm” where the “categories” in question do not exist, but rather “hold” or “obtain.” Accordingly, the basic question concering the ontological status of the ideal (or normative) serves as the volume’s point of departure with Sara Heinämaa’s essay, “Constitutive, Prescriptive, Technical or Ideal? On the Ambiguity of the Term ‘Norm.” In contemporary phenomenology, as Heinämaa says, “the terms ‘norm’ and ‘normative’ are used in several contexts. One dominant argument is that the structure of intentionality is teleological and as such normative” (Heinämaa, 9). Using the examples of being a teacher or a soldier, Heinämaa highlights a difference between two norms. Following a distinction originating in Max Scheler, she notes how there is Tunsollen (“normative ought”), which “implies the concept of rule-following” (Heinämaa, 20) exhibited in customs or social habits. On the other hand, there is Seinsollen (“ideal ought”), a kind of “ideal principle” supplying a constitutive norm involving a “striving for something” (Ibid.) Ideal principles, as Heinämaa observes, “have a constitutive and enabling character: they are not motivational causes for our actions but are conditions that define ways of being” (Ibid.). Crowell further underscores this distinction when, in his reply, he observes that the ideal principles Heinämaa mentions are equivalent to what he means by the term “practical identity” or what Heidegger called a “for-the-sake-of-which” or “ability-to-be” (Seinkönnen); the norm at issue involves a way of understanding oneself, a standard of success or failure exemplified in a felt sensitivity to what is best (or good) given what one is trying to be. Whether we consider being a teacher or a solider, the general point, says Crowell, is that “knowing is something we do in a way possible only for a being who can be guided by a Seinsollen or ideal norm, a ‘minded’ being” (Crowell, 334). Drawing a point that later will become important in the context of Crowell’s understanding of transcendental phenomenology’s relation to metaphysics, he states how, as our knowledge of such ideals is always existential, so it therefore is unsettled and fundamentally unspecifiable. That is just what it means for them to be at issue or at stake in Heidegger’s sense: “Because the ideal that guides what I am trying to be cannot be grounded in truth (fulfillment through Evidenz), it cannot be the topic of a purely theoretical discipline” (Crowell, 333). In doing whatever it is in terms of what one in turn is striving to be, the very ideal of the practical identity itself is at stake, insofar as one’s doing what one does is to work out its meaning, of what it means to live up to it (or not). This is what makes the ideal a measure, and, in the relevant sense, accordingly normative.

Leslie MacAvoy’s essay “The Space of Meaning, Phenomenology, and the Normative Turn,” further clarifies Crowell’s position regarding the normative before going on to criticize the claim that such normativity is imperative to the constitution of meaning. Explaining how the normative turn situates the topic of meaning and validity in relation to the practical norms “for what one ought to do or be” (MacAvoy, 29), she recounts how such an approach thereby characterizes the space of meaning’s purported normativity “in terms of the experience of obligation or binding force” (Ibid.). This normative claim said to underpin meaning, as Crowell has explained elsewhere, amounts to the existential or ontological commitment explaining intentionality and reason: in acting as I do, I always already am implicitly responsible for taking over those “factic grounds” as my reasons. According to MacAvoy, however, phenomenology’s concern with meaning does not entail that such normativity truly plays the role in the formation of meaning that Crowell has argued it does: “While there is a normativity to meaning, it does not consist in the understanding of normativity that has to do with a binding force or claim” (Ibid.). In effect, MacAvoy claims that the phenomenological thesis about the logical, categorial “space of meaning” does not extend to the domain of normativity, as Crowell understands that domain. The binding force of the “ought” does not “capture the normativity of meaning” (MacAvoy, 33). In summarizing the three aspects of Crowell’s characterization of the normative,[1] MacAvoy notes how, for the former, “the norms for whether something can be something are established relative to the norms for doing something” (MacAvoy, 35). By now this will sound familiar. For as Heinämaa had made clear earlier, the very skills and practices in terms of which a thing shows up as what it is are themselves grounded in a practical identity (an “ideal principle”) itself said to be assessible in terms of success or failure. Hence, as MacAvoy says, on the view Crowell defends and which Heinämaa summarizes, the space of meaning bottoms out “in a norm for being a certain type of agent” (Ibid.). This raises the question of the practical identity’s validity, of how such an ideal can be binding, that is to say, of how it can exert a “normative force.” Her main objection is that Crowell’s answer to that question reintroduces the specter of psychologism. Just as psychologism in logic distorts the validity of logic’s content, so interpreting the space of meaning as normative does too, she says. In summarizing MacAvoy’s objection to his position, Crowell writes, “If the normative turn means that phenomenology is a normative discipline, it cannot be fundamental since, on Husserl’s view, all normative disciplines presuppose a theoretical discipline that rationally grounds their prescriptions” (Crowell, 331). If transcendental phenomenology is to be a rigorous science as Husserl envisioned, this appears to entail that it cannot take the normative turn Crowell implies it should, since if it did, so the argument continues, to do so would be to undermine phenomenology’s very claim to theoretical fundamentality to which Husserl took it to be entitled. Before unpacking Crowell’s answer to this concern, it is necessary to further explicate the charge.

To do so, we turn to the question of logic. For if MacAvoy is skeptical as to whether meaning’s categorality is best understood in terms of the bindingness characterizing the existential commitment of practical identity, Walter Hopp’s later essay “Normativity and Knowledge” likewise questions whether the theoretic domain of ideal truth and its connection to knowledge can be understood normatively.[2] Husserl’s phenomenological approach certainly agrees with neo-Kantianism that logical laws cannot be understood empirically, as if they are mere descriptions of how our minds happen to think. The laws of logic are necessary, and hence they are irreducible to descriptive generalizations. And yet at the same time, as Hopp says, for Husserl the laws of logic are not primarily prescriptive judgments regarding how we ought to think, but instead objective in their ideal content and therefore theoretical. Owing to their objective validity, logical laws do have regulative implications for how we ought to think. But that is not their essence. Validity is not the same as normativity.

These disputes concerning the connection between normativity and meaning implicate a more general one that will recur throughout the volume: namely, concern over the relation between transcendental phenomenology and metaphysics. Taking up this metaphilosophical question in “Mind, Meaning, and Metaphysics: Another Look,” Dan Zahavi asks, “Did [Husserl’s] turn to transcendental philosophy, did his endorsement of transcendental idealism, entail some kind of metaphysical commitment, as was certainly believed by his realist adversaries, or did Husserl’s employment of the epoché and phenomenological reduction on the contrary entail a suspension of metaphysical commitments?” (Zahavi, 47).  In Zahavi’s estimation, Husserl’s transcendental turn does not entail the mode of metaphysical neutrality that Crowell contends. As Zahavi concedes, this admittedly is not the dominant view:

“Many interpreters have taken Husserl’s methodology, his employment of the epoché and the reduction, to involve an abstention of positings, a bracketing of questions related to existence and being, and have for that very reason also denied that phenomenology has metaphysical implications” (Zahavi, 51).

Call this widespread reading the “quietist” one. Popular though it is, Zahavi claims that it cannot be correct. Were it true, he suggests, we would be unable to explain why, for instance, Husserl rejected the Kantian Ding an sich and phenomenalism, and why he would obviously have rejected contemporary eliminativism about experience. Even more basically, Zahavi finds the quietist interpretation of Husserl’s transcendental phenomenology as “[running] counter to Husserl’s ambitions” (Zahavi, 50): transcendental phenomenology, says Husserl, as Zahavi notes, is such that there is “no conceivable problem of being at all, that could not be arrived at by transcendental phenomenology at some point along its way” (Ibid.). If transcendental phenomenology’s reduction to meaning involves the kind of radical askesis Crowell maintains, how could Husserl have seen it as being equal to the task of answering every philosophical question we might have? In support of his thesis, Zahavi produces a striking passage from the Cartesian Meditations: “Finally, lest any misunderstanding arise, I would point out that, as already stated, phenomenology indeed excludes every naïve metaphysics that operates with absurd things in themselves, but does not exclude metaphysics as such” (Ibid.) Now as Zahavi acknowledges, passages as these are decisive only to the extent that we clarify the term “metaphysics,” which notoriously is ambiguous. He proposes at least three senses it can mean in Husserl: first, a theoretical investigation of fundamental reality (Zahavi, 51); second, philosophical engagement with questions as “facticity, birth, death, fate, immortality, and the existence of God” (Zahavi, 52); third, reflection on the status of “being and reality” (Ibid.). But here, the crucial caveat Crowell rightly mentions in reply must be noted: although Husserl claims that so-called metaphysical questions retain their sense, that is so only “insofar as they have possible sense in the first place” (Zahavi, 51). Accordingly, then, the task becomes one of determining the limits of sense, of what is open to phenomenological Evidenz, and what is not.

If Crowell on one front must defend his conception of radical askesis from the charge that it neglects the metaphysical implications of phenomenology, he must also on the other face challenges from a series of articles by Mark Okrent, Glenda Satne and Bernado Ainbinder, and Joseph Rouse, which, taken together, aim to undermine the transcendental thesis that meaning and normativity are irreducible to nature. As for Okrent, he levels two objections, the first of which, amounting to a reformulated version of the infamous “decisionist objection” to Heidegger’s conception of Angst, Crowell dispatches quickly. As for the second, it contends that there is no way of truly understanding the human mode of being-oneself as a normative achievement whose form is different from “our animal cousins” (Okrent, 173). But as Crowell responds, even if one grants that animals do in some relevant sense act in accord with norms, they do not act in light of them—not only do they lack language so as to be answerable to others for their actions as we are for ours, they also are unable to measure themselves in a non-representational way that is responsive to an intelligible sense of what is best. Even in the simplest forms of perception, the domain of sens sauvage Charles Siewert calls “recognitional appearance,” there is an element of normativity at work marking the experience as distinctively human. In seeing as we do, experience as Siewert notes is “as much in the service of imagination as of judgment, and integral to the activity of looking, which is subject to norms of its own” (Siewert, 290). That this distinctively human richness to experience is lacking in other creatures is evident in that, just as they do not dwell in a “world” wherein things are inflected by the claim of a “good beyond being” (a claim concerning how things ought to be), so they only inhabit an “environment,” a surrounding in which beings are what they are, and nothing more. Satne’s and Ainbinder’s proposal—to “put agents back into nature”—which in turn aims to rehabilitate a “relaxed naturalism,” accordingly takes issue with this conclusion. Contrary to Crowell’s transcendental approach, the aim is to ground normativity in the contingent features of life “shared with other animals.” Joseph Rouse’s essay which contends for a “radical naturalism” joins the cause, seeking to explain our normative capacities in terms of our biology. In defending what he sees as a methodological gap between transcendental phenomenology and empirical science, Crowell reverts to a powerful strategy: according to him, these kinds of attempts to naturalize meaning and normativity require a construction—in this case, “life” —that “transcends the kind of Evidenz to which transcendental phenomenology is committed” (Crowell, 347). Further, because such accounts make use of empirical beings, they are “ontic” and therefore metaphysical in precisely the sense Crowell’s notion of reductive askesis forbids.

Presuming Crowell is correct that transcendental phenomenology establishes why nature cannot explain normativity and thereby fails to ground meaning, what then is the source of normativity? In reply, here one might to choose follow Husserl’s path as John Drummond does, maintaining that intentionality as such (and hence normativity along with it) is governed by a rational telos. As he says, “Husserl believed that in all three rational domains—the theoretical, the axiological, and the practical—the aim of experiential life is the same: to live a life of intuitive evidence, to live the life of a truthful, rational agent” (Drummond, 110). Just as intentionality is structured by the norm of intuitive fulfilment, so we are beings whose form of life involves a kind of rational self-responsibility that remains inexplicable on naturalistic terms. Drummond’s essay concludes by stressing what he takes to be a great merit of his account’s view of self-responsible convictions, namely its easy ability to also account for moral—or ethical—normativity. The issue of practical normativity with which Drummond’s contribution ends is taken up through the lens of Levinas’s relation to Kant in the volume’s next essay, Inga Römer’s “The Sources of Practical Normativity Reconsidered—With Kant and Levinas.” Contrary to what a reading limited only to Levinas’s early thinking may suggest, Levinas finds Kant’s philosophy of practical reason congenial to his own mission of exploring the ethical implications of a good beyond being. As Römer comments, Kant’s notion of disinclination can be seen as a relative to what Levinas himself characterizes as the an-archic and rational desire for the infinite. To see the two’s similarities, however, is not to deny their important differences. Römer lists three, the most significant perhaps of which is that, while it is not entirely misleading to name Levinas’s thinking “a philosophy of heteronomy,” there is a sense in which the self becomes truly autonomous due to “the signifying call of the Other” (Römer, 123). After further unpacking the Kantian position through an analysis of Christine Korsgaard’s notion of practical normativity, Römer then recounts Crowell’s Heideggerian criticism of it, finally to formulate an objection against Crowell’s view of reason-giving as constitutive of the second-personal ethical stance. The concern is that a trace of egoism still remains: “Even if I am required to give an account of my reasons to others, does such an account not tend toward a certain ethical self-conceit? If I am the ultimate source of measure, even if I need to defend this measure with respect to others in order to not contradict myself by taking my reasons to be private ones, does this view not place the self at the center of ethics?” (Römer, 131). Concern that the Heideggerian approach to practical normativity cannot eliminate all residue of self-conceit is well-founded. But while Römer takes Levinas’s own approach to avoid such a pitfall, one may wonder whether it does. When she remarks, for example, that in Levinas’s view there is “no God beyond ethical significance that would be the source of ethical normativity” (Römer, 126), does not the threat of self-conceit arise once again? Even if the asymmetry said to define one’s encounter with the other suffices to annihilate a kind of egoism, does it purge the least trace of it? For the total annihilation of self-love Levinas claims to be seeking, one might argue that only an encounter before God is truly sufficient.

Returning to the question of meaning’s source left hanging in the debate between Drummond and Crowell, Irene McMullin for her own part leans towards a view closer to the latter’s own, preferring a Heideggerian approach in which both meaning and the normative are said to be ungrounded—ultimately, says McMullin, there is no forthcoming answer as to how we find ourselves immersed in a meaningful world. We simply do, and that we find ourselves so situated is a reason for gratitude. Thus, as she says, although “resolute Dasein” experiences the “dizzying, disorienting sense of panicked terror” (McMullin, 149) accompanying the felt realization of meaning’s groundlessness, that realization is followed by another, the “incredible sense of relief and gratitude”(Ibid.) that there is any meaning at all, however ungrounded and contingent it is. This gratitude in turn resolves us to “love better, to strive more fully, to treat the goods in our lives more tenderly” (McMullin, 150). If McMullin’s analysis of the role of receptivity in resoluteness is a welcome corrective to the tendency to see authenticity in overly heroic or active terms, Joseph K. Schear’s essay, “Moods as Active,” does well to correct for an error arising from the opposing tendency of viewing moods as purely passive. Not only are moods an expression of agency, says Schear, they are structured normatively insofar as they are responsive to intelligible interrogation (by others and ourselves). As he notes, it is far from committing a category mistake for someone to ask of us why we are feeling as we are. Interrogating a mood is fair game. While we cannot choose our moods as we choose to make up our minds about what to believe, neither are moods always experiences in which we are just passive. Against a consensus that sees moods as “closer to sensations than judgments” (Schear, 220), he notes that moods do not arrive like “a hurricane, or the fog” (Ibid.). They are episodes in which we may intervene. A mood is an item we can manage, whether by trying to escape it through replacing it with another one, distracting ourselves from it, or by conspiring with it so as to feed and prolong it. However, ultimately the kind of agency interesting Schear is not the preceding kind of “agency over our moods” (Schear, 221), but the expression of agency in it. This second sense of agency is present in moods, he argues, precisely to the extent that we are able to answer intelligibly to the mood-question: “Why are you anxious?” or “Why are you joyful?” Such answerability, so he concludes, is due to being in a mood’s involving one’s living it out as a “responsive orientation to one’s situation.” In a contribution complementing Schear’s well, Matthew Burch in “Against our Better Judgment” explores the phenomenon of akrasia. There is much to be said for this very rich and thoughtful selection, but perhaps most noteworthy is its phenomenological clarification of the notion of “interest,” a middle category between brute desire and explicit judgment or commitment. Interests, hence, are meaningful affections, “things we care about” and things “in which we have a stake” (Burch, 233). Though Burch goes on to develop the notion of interest into a wider account of how a Heideggerian conception of authenticity answers to how norms bind us, with an eye toward concluding the review, here I should like to take Burch’s discussion in a slightly different direction: what is our interest in doing phenomenology? What exactly calls us to it, and what guides and sustains its commitment?

To answer these questions is to return to Crowell’s understanding of phenomenological philosophy’s role in the task of clarifying meaning—here, specifically the task becomes making sense of the very one who philosophizes in the way its normative turn proscribes. As has become clear in assessing Crowell’s response to his critics, the notion of askesis is the cornerstone to his approach. According to him, the reduction to meaning entails that transcendental phenomenology neither demands (Heidegger) nor entails (Husserl) a metaphysics to complete itself. Thus, his own position parts ways with both Husserl and Heidegger. As he observes in his concluding essay, as the question of transcendental phenomenology’s relation to metaphysics “constitutes the horizon of transcendental phenomenology, so I will conclude by considering it under three closely related headings: naturalism, metaphysics, and theology” (Crowell, 345).

Taking the measure of things (we ourselves above all not excepted) in its distinctive fashion, Crowell’s notion of transcendental phenomenology is a philosophy of enigma. What can be intuited in the light of Evidenz is clear and distinct, while anything beyond is consigned to antinomy. The situation accordingly comes to one of deciding how to understand where transcendental phenomenology draws the limits of intuitable meaning. Where precisely does the threshold lie? And what about the meaning, if any, lying beyond the threshold separating what is given in genuine first-person evidence from what is not? Is such meaning to be set to the side, or must not it somehow be integrated into the existence of the one who encounters it? If it must be integrated, how is that task of existential incorporation to be coordinated in terms of the norm of reductive askesis which, qua phenomenologist, entails bracketing such meaning? There looms, so it would seem, a fissure in the being of the one thinking phenomenologically. To begin with, as just noted, there is the difficulty of deciding what does (and thus does not) lie within the bounds of intelligibility. To decide with Crowell that we ought to refrain from taking a phenomenological stand on anything beyond the intuitable is a mark of intellecutal humility, to be sure. Nobody should deny that it is advisable to suspend judgment when things are sufficiently ambiguous. Yet such a suggestion remains formalistic; it cannot resolve how we are to apply it. How, then, are we to determine when not making a commitment in the face of the meaning at issue is truly the humble and rationally reponsible thing to do? To be confident in a given situation that we are doing what humility dictates implies that we are entitled in judging that what before us seems less than self-evident is in fact as obscure as it appears. How, however, are we to know that we are correct, that we are justified in that stance?

It is not an uncommon experience in life to come to learn that something we initially thought was unclear actually was not; the unclarity resided in our vision and not the thing. It is was not that the thing was veiled, just that we were failing to see. Hence, while it is good to be duly skeptical of claims that make genuiniely ungrounded claims of metaphysical speculative excess (“Everything is illusion, for we are in a quantum simulation!”), we should be mindful that determining when that is so can itself be fraught; something could in principle be grounded in evidence even if, or when, it does not yet seem so to us. Anyone who is honest will admit that there are reasons for thinking that the judgments we reach based on what we believe is humility can turn out instead to have been motivated by a subtle pride or stubborness. It is important to note, for instance, that this strand of epistemic humility is for all its virtues only partial; it essentially is an intellectual askesis. Or more exactly, insofar as it is it supposed to be an effort of epistemic self-discipline, it begins to undermine its own spirit of modesty the moment it slips into more than that by coming to resemble more so a general posture toward the whole of existence. When that happens, one important norm governing our trafficking in meaning is elevated to something instead approaching an absolute. And it is not difficult to see how, in doing so, it can propel the one who treats it in that way along his own path of error and blindness. This becomes more apparent when we consider another of humility’s aspects: namely, humility’s willingness to yield to things by accepting something for what it is, thereby submitting to the disclosed. Crowell’s reductive askesis, along with its norm of epistemic humility, arguably threatens to imperil an authentic encounter with meaning so interesting it if absolutized to trump all else. A commitment to the norm of truth-seeking, for example, may at times require passing beyond what presently appears to be grounded in evidence. Life presents us with these situations constantly. We resolve the indeterminateness by commiting to a course of judgment or action despite the ambiguity. Just as the meaning of some situations becomes clear only in retrospect years later or in an unanticipated flash of insight, so some truths become evident after a period in which they had not been. To refuse to commit to taking a stand on something that remains less than intuitively clear means what might have crystalized never will. If, then, humility is not synthesized with other considerations (including trust, hope, patience, wisdom, or courage), it threatens to constrict rather than expand meaning.

Insofar as the reductive askesis of Crowell’s position ends with enigma, it has been my suggestion that such enigma potentially implicates more than what that methodological stance admits. Meaning by its nature implicates our having to take a stand on what lies on the margins of intentionality, what at any moment makes itself felt as an unspecifiable more. To ignore this surplus of sense in the name of a humility that does not take a stand on what it sees as undecidable is to neglect precisely what puts our existence at stake and at risk in the first place. The respective imperatives of the transcendental (epistemic askesis) and the ontological (existential commitment) appear to be tugging in opposite directions. As a philosophy of meaning, transcendental philosophy can attempt to delimit the things whose meaning we are said to be justified in taking a stand on philosophically. In refusing to take a stand on what it considers the metaphysical, however, this gesture of refusal only implicates the omnipresence of that something more—that excess—with which we all must grapple existentially. Thus, while we may have reached the limits of what a certain mode of intuitive thought can decrypt, it does not follow that it has thereby established the bounds of the meaningful as such. Could not more remain to be given?

Here a detail concerning the earlier debate between Zahavi and Crowell over transcendental phenomenology’s relation to metaphysics will be recalled. Zahavi mentions that, taken in its second sense, metaphysics for Husserl deals with matters of birth, death, fate, immortality, and the existence of God. Do these metaphysical questions have a possible sense? What is their relation to transcendental phenomenology, as Crowell understands it? For his own part, Crowell states that while a “phenomenology of faith” is possible, it does not disrupt any of the essential metholodological commitments of transcendental phenomenology. For Crowell, that means rejecting a traditional commonplace according to which revelation is said to complete what reason cannot. Whatever room it leaves for faith, it must not interfere with the autonomy of a presuppositionless phenomenological reason.

This expulsion of faith from the project of transcendental phenomenology is, in a way, simply a specific application of the general reduction from entities to meaning. As Crowell says, “Transcendental phenomenology is not concerned with entities at all” (Crowell, 337). But if transcendental phenomenology is not concerned with entities, what about the entities that we ourselves are? How does the normative turn handle la question du sujet? Because such an approach seemingly comes up lacking, it calls for another inquiry to accomplish what it cannot. Transcendental phenomenology, after all, can trace the general contours of existence, telling us that we should live a self-responsible existence in light of the rational norm of evidence. It can clarify what it most generally means to be in the space of meaning. But it cannot decide where the limits between sense and nonsense lie in a given case, nor what precisely living up to the norm of evidence entails in any or every particular instance. We can reflect on the general normative structures of existence and how those structures make an encounter with entities possible, yet ultimately life still must be lived. For Crowell, perhaps much of what we take at a first-order level to be meaningful is not. Or at least the meaning in question falls short of Evidenz, in the transcendental sense. The things we take ourselves to know, on closer inspection, really are a matter of antinomy. This would be true for the theological, for in giving a name to what it takes to have addressed it, it does so without sufficient evidence. As Martin Kavka says in characterizing banal theologizing, “This argument entails the claim that the problem with any and all theologization of phenomenology is that theology determines” (Kavka, 92). Or as Crowell puts it, by trying to give a name to the anonymous claim that has addressed it, such a response crosses into antinomy.

Antinomy is also the figure which best describes the third horizon of transcendental phenomenology, the “theological turn,” in which phenomenology abandons reductive askesis to posit a prior condition of correlation, variously called “event,” Erscheinung als solches, “givenness,” “phenomenality,” or “revelation” (Crowell, 349).

Where, then, does this leave man’s search for meaning, and the question of his destiny? Concluding with a provocative but basic question does well to underscore the exigency of the methodological situation’s existential import. What, in short, are we to make phenomenologically of the claim that Jesus Christ is the Son of God? Does the claim fall within phenomenology’s remit? Outside it? Is it essential to phenomenology’s venerable aim of putting oneself in question, or orthogonal to that attempt? Is to affirm such a claim consistent (or not) with the promise of meaning, as Crowell understands that promise? One must make the decision to leap—or not. Either way, a decision is made. Seen strictly from the perspective of a transcendental critique of meaning, what faith claims to see—namely, that because Jesus Christ is the way, the truth, and the life, he is the absolute measure by which our own individual existences are to be measured—probably will be viewed as an affirmation having succumbed to “metaphysical” tempation. It views faith as epistemic folly. But assuming this is what a transcendental phenomenology’s critique of meaning entails, why not then see it as a reductio of that position?

To succeed in its aim of clarifying meaning, cannot it be suggested that transcendental phenomenology must be understood not as deeming what faith sees as inscrutable, but rather as itself calling for it?[3] To rest content with phenomenological askesis is to leave ourselves in a state of unnecessary indeterminateness. As it does not countenance the mysteries of God, so for it the enigmas of human existence find no solution.[4] It is thus left to face a potential incoherence of its own approach. The latent incoherence is manifest methodologically insofar as it fails to make intelligble what could be made so were it to complement its vision with what its own commitment to the imperative of self-givenness implicates. By not doing so, it ends in a failure of sense-making. For this reason, arguably it can be considered a failure relative to its own internal aim of trying to clarify meaning. The lack of success is most evident when seeing its inability to make adequate sense of ourselves.

We are finite creatures, and so meaning is finite. We can grasp the world as it is, though never as a whole; and if there is anything beyond that, it is a matter of faith, not philosophy. A phenomenology of faith is certainly possible, but transcendental phenomenology cannot be said to be exceeded by something that escapes it and yet grounds it, such that a “theological”—or “naturalistic” or “speculative” or “metaphysical”—turn is required. One who nevertheless wishes to make such a turn must show why it does not end in antinomy, the “euthanasia of pure reason” (Kant 1998, 460). Take your pick: deus sive natura; mereological universalism or nihilism; “neutral monism”; panpsychism; flesh, life, desire. In the face of antinomy, the askesis of transcendental phenomenology is not egoism but modesty, not a “theory of everything” but a clarification of what matters. Its claim on our “ultimate philosophical self-responsibility” (Husserl) is irrevocable if we are committed to having evidence for what we say. Just this defines the normative turn from entities to meaning, the promised land, “the secret yearning of all modern philosophy” (Crowell, 352).

By addressing the “question of the subject” in a way that entails no answer is ever forthcoming, reductive askesis renders the need for putting ourselves into question otiose, even futile. The misunderstanding at work in its approach to the entities that we ourselves are is seen precisely in its failing to live up to its own impulse to truly make sense of our existence. Here, in short, would be a philosophy concerned with explicating the meaning of things while simultaneously failing to ground its own existence in any firm meaning. If, in fact, there ultimately was no true meaning to existence because there were no answers to life’s ultimate questions, why then should a philosophy about meaning try to make sense of that meaning? To be sure, doing so could still serve as an idle pursuit perhaps, as a way for those so inclined to pass the time. But philosophy must be more than intellectual tiddlywinks. For were it not more than that, what reasons do others have for caring about what such a philosophy says? Philosophy would not just lose its exigency, but its universality too. In the last analysis, any philosophy of meaning stifling the yearning for the absolute does so on pain of compromising the coherence of its express aim. In restless pursuit of a meaning it cannot find, its is a critique of meaning that renders human existence as if it ultimately had none. Reaching only a mirage of the true promised land, as with Dathan it dies in the wilderness.

Bibliography

Husserl, Edmund. Cartesian Meditations: An Introduction to Phenomenology. Translated by Dorion Cairns. The Hague: Martin Nijhoff, 1973.

Husserl, Edmund. The Crisis of European Sciences and Transcendental Phenomenology: An Introduction to Phenomenological Philosophy. Translated and introduction by David Carr. Evanston, IL: Northwestern University Press, 1970.


[1] With Crowell, MacAvoy notes, first, the distinction originating in Husserl’s Phenomenology as Rigorous Science between the empirical and transcendental (that is, between the natural and normative); transcendental consciousness, MacAvoy explains, is governed by a lawfulness other than the causality of the psychical and physical. Second, and relatedly, the intentional experience of an object involves a command over its “implications,” the inner and outer horizons of sense in terms of which the object itself can be taken as what it is, the paradigmatic example being the perceptual object, since, say, in perceiving a cube, I must “co-intend” its sides that are not directly seen but are nevertheless implicated. Perceptual intentionality accordingly has a “motivational” logic: If I were to move here, then the cube’s other sides should come into view.

[2] In this way, Hopp’s essay follows in the footsteps of his mentor, the late Dallas Willard, whose early works on Husserl’s view of logic, ideality, and the possibility of knowledge remain exemplary. See, for example, Logic and the Objectivity of Knowledge: A Study in Husserl’s Philosophy (Ohio University Press, 1984).

[3] For a comprehensive analysis of God’s role in Husserl’s transcendental philosophy, see Emmanuel Housset’s Husserl et l’idée de Dieu (Paris: Cerf, 2010). For a critical appraisal of Husserl’s attempt to incorporate God into his transcendental approach, see John Drummond’s draft paper, “Phenomenology, Ontology, Metaphysics,” The Boston Phenomenology Circle, Accessed September 18, 2019.

[4] If this language is recognizably Blondelian, it is because Maurice Blondel’s own thought systematically explored the reciprocal interface between reason and revelation, philosophy and faith. For an explanation of how philosophy’s concern over the enigma of existence implicates a fulfillment in the mysteries of God, see Jean Lacroix’s short study Maurice Blondel: An Introduction to His Philosophy, trans. John C. Guiness (Sheed and Ward: New York, 1968), 64-66.